You are on page 1of 41

TYPE BREAK UP OF QUESTIONS

No. of
UNIT Topic
Questions
Indus Valley Civilization: Features, Sites, Society, Cultural History, Art and
5
Religion
Vedic Age 6
Mahajanapadas 2
Religions-Jainism and Buddhism. 7
The Maghadas and the Mauryan 4
Foreign invasions on India and their impact, the Kushans and Miscellaneous 4
The Sathavahanas 1
The Sungas 2
The Gupta Empire -their Administration- Social, Religious and Economic
7
conditions-Art, Architecture, Literature, Science and Technology.
South Indian Dynasties - The Badami Chalukyas, the Eastarn Chalukyas, the
Rastrakutas, the Kalyani Chalukyas, the Cholas, the Hoyasalas, the Yedavas, the 18
Kakatiyas and the Reddis.
The Delhi Sultanate-Administration, Economy, Society, Religion, Literature,
5
Arts and Architecture.
The Vijaynagar Empire 3
Mughal Empire 5
the Bhakti Movement and Sufism 5
The European Trading companies in India- their struggle for supremacy-with
special reference to Bengal, Bombay, Madras, Mysore, Andhra and Nizam, 15
GovernorGenerals and Viceroys.
Indian War of Independence of 1857 - Origin, Nature, causes, consequences
1
and significance with special reference to Concerned State
Religious and Social Reform Movements in 19th century in India and
4
Concerned State
India's Freedom Movement 9
Revolutionaries in India and Abroad. 1
Mahatma Gandhi, his thoughts, Principles and Philosophy. Important
6
Satyagrahas,
the Role of Sardar Patel, Subash Chandrabose in Freedom Movement 2
Post - independence consolidation. Dr. B.R. Ambedkar, his life and contribution
3
to making of Indian Constitution
India after Independence - Reorganization of the States in India. 3
GRAND TOTAL 120

AMIGOS IAS 1
APPSC – GROUP-I, Prelims – TEST 15 (14-02-2024)
1. With respect to Indus valley civilisation,  Dharmasutras are religious texts written in
Dilmun and Makan are: Sanskrit by Brahmins. They are guidebooks
1) Coins 2) Trading stations that contain guidelines for social and
3) Seals 4) Traditional clothes individual behavior, ethical norms, and
Answer: 2 personal, civil, and criminal law. They also
Explanation: discuss the rights and duties of people at
different stages of life.
 The Mesopotamian texts speak of two
intermediate trading stations called Dilmun  Sulvasutras are a collection of geometrical
and Makan, which lay between constructs that are thought to have been
Mesopotamia and Meluha. created around the end of the Vedic period.
Hence all the statements are correct.
 Dilmun is probably identifiable with Bahrain
3. The term Syama-ayas during the later Vedic
on the Persian Gulf. Hence Option (2) is
period means:
correct.
1) Bronze 2) Iron
2. Consider the following statements regarding
Sutra literature: 3) Gold 4) Carpenters
I. Srautra Sutra: Deals with large public Answer: 2
sacrifices. Explanation:
II. Griha Sutra: Deals with rituals connected  Iron was an important metal used for
with birth, marriage, naming, funerals, implements in the later Vedic period.
etc.  It was called syama-ayas or krishna-ayas, or
III. Dharma Sutra: Explains social and local dark metal. Hence, option (2) is correct.
customs.  Iron is believed to have played an important
IV. Sulva sutra: Prescribe various kinds of role in the conversion of the forests of the
measurements for the construction of Ganga Valley into agricultural lands.
sacrificial altars. 4. Siksha, Kalpa, Vyakarna, Nirukta, Chhanda,
Which of the above given statements is/are and Jyotisha are classified under which one of
correct? the following?
1) I and II 2) I, II and III 1) Vedangas 2) Brahmanas
3) I, II and IV 4) I, II, III and IV 3) Aranyakas 4) Upanishads
Answer: 4 Answer: 1
Explanation: Explanation:
 The Srauta Sutras are part of the Sanskrit  Six vedangas (limbs of the Vedas) were
sutra literature corpus. Their contents, evolved for the proper understanding of the
which are similar to those found in Vedas.
the Brahmana levels of the Vedas but given  These are Siksha (phonetics), Kalpa (rituals),
in a more systematic and precise manner, Vyakarna (grammar), Nirukta (etymology),
include instructions pertaining to the Chhanda (metrics), and Jyotisha
employment of the Sruti corpus in significant (astronomy).
rites and the right conduct of these big Vedic  Each vedanga has developed credible
ceremonies. literature around it, which is in sutra form,
 The Grihya Sutra contains rites for i.e., precepts. Hence, option (1) is correct.
exceptional events, seasonal ceremonies,  This is a very precise and exact form of
and five daily sacrifices, notably from the expression in prose that was developed by
beginning of life to the end, i.e. death. the ancient Indians.

AMIGOS IAS 2
 Panini's Ashtadhyayi, a book on grammar in Megasthenes states that he did not notice
eight chapters, is the final culmination of this any slaves in India. But there is no doubt that
excellent art of writing in sutras (precepts), domestic slaves were found in India from
in which every chapter is precisely Vedic times onwards.
interwoven.  For the first time in the Maurya period,
 Besides the Vedas, the Brahmanas, the slaves were engaged in agricultural work on
Aranyakas, and the Upanishads are also a large scale. The state-maintained farms, on
included in the Vedic literature and are which numerous slaves and hired labourers
known as the later Vedic literature. were employed. Hence statement II is
5. Which of the following Jain texts describes 16 correct.
Mahajanapadas?  Kautilya advised that new settlements
1) Bhagwati Sutra 2) Kalpasutra should be founded with the help of
3) Parishishtaparvan 4) Anguttara Nikaya cultivators, who were apparently Vaisyas,
Answer: 1 and with that of Sudra labourers, who should
be drafted from overpopulated areas.
Explanation:
The Bhagwati Sutra contains information  In order to bring the virgin soil under
cultivation, the new peasants were allowed
about Jain cosmology, philosophy, and
ethics. It also describes the 16 remission in tax and supplied with cattle,
Mahajanapadas, which were the 16 most seeds, and money. The state followed this
powerful and prosperous kingdoms in ancient policy in the hope that it would get back
India. Hence, option (1) is correct. what it had given. Hence statement III is
correct.
6. Consider the following statements with
respect to the state of the agricultural system 7. In the context of the ancient history of India,
in the Mauryan period: Panyadhyaksha is related to:
I. Mauryan State provided irrigation 1) Navy 2) Tolls
facilities and regulated water supply. 3) Commerce
II. The Mauryan state employed slaves in 4) Weights and measurements
agriculture. Answer: 3
III. To bring the virgin soils under cultivation, Explanation:
the new peasants were allowed remission Panyadhyaksha was an official in the Mauryan
in tax and supplied with cattle, seeds, and Empire who was responsible for trade, price
money. fixation, and the sale of goods produced by
Which of the statements given above is/are state-run manufacturing units. Hence, Option
correct? (3) is correct.
1) I only 2) I and II only 8. Consider the following statements with
3) II and III only 4) I, II and III respect to the economic conditions of the
Answer: 4 Guptas:
Explanation: I. Brigukachchaha, Kalyana, and Sind were
the main trade centres with the Romans.
 The Mauryan state also provided irrigation
II. Ujjain was a major commercial Center.
facilities and regulated water supply for the
benefit of agriculturists. Megasthenes Which of the statements given above are
informs us that in the Maurya Empire, the correct?
officer measured the land as in Egypt and 1) I only 2) II only
inspected the channels through which water 3) Both I and II 4) Neither I nor II
was distributed into smaller channels. Hence Answer: 3
statement I is correct.  Explanation:
 The striking social development of the Agriculture was the main occupation in the
Maurya period was the employment of Gupta Empire, and there was no
slaves in agricultural operations. governmental interference. The land was

AMIGOS IAS 2
fertile, and the means of irrigation were  Many Indian gods and goddesses are
simple. depicted on Kushana coins, along with many
 The trade contacts developed during the Persian and Greek deities.
Kushana Period continued, and  The Gupta gold coins, known as Dinars, are
Chandragupta II’s conquest of western India among the finest examples of numismatics
further added to this trade. The people were and aesthetic excellence.
prosperous, and they were free to grow and  The governing king and legends were shown
flourish. The important port towns on the front of the coins, while a goddess
were Brigukachchaha, Kalyana, and Sind, was depicted on the reverse. Hence
which were bulk trade centres with the statement II is correct.
Romans. Ujjain had become a major  Pre Guptan and Guptan Coins
commercial center, and it was linked to
 The Gupta monarchs were known for their
southern and northern India. Nasik, Paithan,
gold coins, and some academics refer to the
Pataliputra, and Benares were other major
Gupta dynasty's rule as the "rain of gold"
trade centers. Hence statements I and II are
because of the abundance of gold coins.
correct.
 The Satavahanas issued coins of lead and
 The Gupta period had many cloth centers,
potin (base silver).
and the silk industry witnessed a significant
development during this period. The  Satavahanas’ copper coins with the symbol
Mandsor Inscriptions give account of the fact bearing a ship were abundant in
that the Gupta people were helped to a great Avanti. Hence statement III is correct.
extent by the growth of the silk industry. 10. Consider the following statements regarding
Gold, silver, and copper were used in making Harappan trade:
ornaments and issuing coins. The gold coins I. The Harappans used metal money in
show the pomp, power, and prosperity of trade.
the empire. II. They set up a trading colony in northern
9. Which of the following statements is/is Turkmenistan.
correct about post-Mauyran coinage? III. Harappans have enough of the necessary
I. The Kushans were the first kings to issue raw materials for the commodities they
the largest cache of gold coins in India. produce.
II. The Gupta coins contained the portrait or IV. They had commercial links with the
bust of the king on the front side, and in people of the Tigris and Euphrates basins.
the back, a deity was depicted. Which of the statements given above is/are
III. Satavahanas’ copper coins with the incorrect?
symbol bearing a ship were abundant in 1) I and II 2) I and III
Avanti. 3) I, II and III 4) I, II and IV
Select the correct answer using code below: Answer: 3
1) I and II only 2) II and III only Explanation:
3) I and III only 4) I, II and III • The Harappan cities did not possess the
Answer:4 necessary raw materials for the
Explanation: commodities they produced. They did not
 Kushanas issued mostly gold coins and use metallic money. We have no idea
numerous copper coins, which have been about their currency. Most probably, they
found in most parts of North India up to carried on all exchanges through barter in
Bihar. The Kushanas were the first Indian return for finished goods and possibly
kingdom to mint gold coins in large foodgrains. Hence statement I and III are
quantities between the 1st and 4th centuries not correct.
CE. Hence statement I is correct. • Their cities carried on commerce with
those in the land of the Tigris and
Euphrates. Many Harappan seals have

AMIGOS IAS 3
been discovered in Mesopotamia, and it  Trephination implies the surgical process of
seems that the Harappans imitated some drilling holes in a damaged skull to remove
cosmetics used by the urban people of any bits of bone from a fractured skull and
Mesopotamia. Hence statement IV is then cleaning out the blood pooling under
correct. the skull after an injury to the head.
• They set up a trading colony in northern  The fact that Trephination was known to the
Afghanistan, not in northern people of the Indus Valley civilization has
Turkmenistan. Hence statement II is been attested to by evidence from three
incorrect. sites, namely, Lothal, Burzahom, and
11. Consider the following statements: Kalibangan. Hence, Statements I and IV are
I. Asthadhyayi was written by Patanjali. correct.
II. Sarvavarman, a scholar of great repute,  At Lothal, a skull of a child aged around 9–10
composed the grammar of the Katantra. years, which had a squarish hole on the right
III. Hala wrote a great poetical work, temporal side, was found.
Gathasaptasati, in Sanskrit.  Archaeologist Roy Chowdhury stated that
Which of the statements given above is/are one skull obtained from Kalibangan in
correct? western India was to be trephined as well.
1) I only 2) II only 13. With reference to ancient history,
3) II and III only 4) I and III only “Kutagarashala” literally means:
Answer: 2 1) A place where animals were kept.
Explanation: 2) a place where intellectual debates among
Buddhist mendicants took place.
 The most outstanding work in the field of
grammar, Mahabhasya, written by Patanjali 3) A place where weapons were stored.
in the second century B.C., is a commentary 4) A place to sleep.
on Panini's Asthadhyayi.Hence statement I is Answer: 2
not correct. Explanation:
 Sarvavarman, a scholar of great repute in the  Kutagarashala – literally, a hut with a pointed
court of the Satavahana King Hala, roof – or in groves where travelling
composed the grammar of the Katantra. This mendicants halted.
work was short and handy, which helped me  Debates between philosophers took place
learn Sanskrit in about six months. Hence here.
statement II is correct.  If a philosopher succeeded in convincing one
 Hala wrote a great poetical work, of his rivals, the followers of the latter also
Gathasaptasati, in Prakrit. Hence statement became his disciples.
III is not correct. 14. Which of the following factors was mainly
12. Evidence of trephination as a remedy for responsible for the rise of the Magadha
certain diseases comes from some of the empire?
following: I. Availability of iron mines
I. Lothal II. Use of elephants in military
II. Harappa III. Orthodox character of Magadhan society
III. Daimabad Select the correct answer using the code given
IV. Kaibangan below.
Choose the correct answer from the following 1) I and II only 2) II only
options: 3) II and III only 4) I and III only
1) II and IV 2) I, II and IV Answer:1
3) III and IV 4) I and IV Explanation:
Answer: 4  Magadha enjoyed an advantageous
Explanation: geographical position in the age of iron
because the richest iron deposits were

AMIGOS IAS 4
situated not far away from Rajgir, the earliest  It forms, in context, a continuation of his
capital of Magadha. Hence, Statement I is other drama, Pratijnayaugandharayana.
correct. 16. With reference to the religious history of
 Magadha enjoyed a special advantage in India, consider the following statements:
military organization. Although the Indian I. Sautranika and Sammitiya were the sects
states were well acquainted with the use of of Jainism.
horses and chariots, it was Magadha that II. Sarvastivadin held that the constituents of
first used elephants on a large scale in its phenomena were not wholly momentary
wars against its neighbours. The eastern part but existed forever in a latent form.
of the country could supply elephants to the Which of the statements given above is/are
princes of Magadha, and we learn from correct?
Greek sources that the Nandas maintained
1) I only 2) II only
6,000 elephants. Hence, Statement II is
3) Both I and II 4) Neither I nor II
correct.
Answer: 2
 The unorthodox character of Magadhan
society. It was inhabited by the Kiratas and Explanation:
Magadhas, who were held in low esteem by  Sautranika and Sammitiya are Buddhist
the orthodox Brahmanas. Hence statement sects. Hence, statement I is incorrect.
III is incorrect.  The Sarvāstivāda school believes that all
15. 'It is a Sanskrit play written during the Gupta things exist continuously, in the past,
period. It contains romantic narratives about present, and future. Hence, statement II is
a Vatsa king and the daughter of a ruler of correct.
Avanti. The main theme of the drama is the 17. Consider the following statements with
sorrow of the king for his queen, believed by respect to the Dancing Girl sculpture:
him to have perished in a fire.' I. It is the world's oldest bronze sculpture
Which of the following texts is being described found in Harappa.
in the above-given passage? II. It is a four-inch figure that depicts a naked
1) Mrichchhakatika girl wearing only ornaments.
2) Meghadutam III. She stands in a tribhanga posture with her
3) Swapnavasavadattam left hand on her hip.
4) Ratnavali Which of the above given statements is/are
Answer: 3 correct?
Explanation: 1) I and II 2) II and III
 Swapnavasavadattam is a Sanskrit play in six 3) II only 4) III only
acts written by the ancient Indian poet Answer: 3
Bhasa. Hence, option (3) is the correct Explanation:
answer.  The dancing girl is the world’s oldest bronze
 It was written during the Gupta period. sculpture found in Mohenjodaro.Hence
 The plot of the drama is drawn from the statement I is incorrect.
romantic narratives about the Vatsa king  This four-inch figure depicts a naked girl
Udayana and Vasavadatta, the daughter of wearing only ornaments, which include
Pradyota, the ruler of Avanti, which were bangles on the left arm and an amulet and
contemporary to the poet's time. bracelet on the right arm.Hence statement II
 The main theme of the drama is the sorrow is correct.
of Udayana for his queen Vasavadatta,  She stands in tribhanga dancing posture with
believed by him to have perished in a fire, the right hand on her hip. Hence statement
which was actually a rumour spread by III is incorrect.
Yaugandharayana, a minister of Udayana, to 18. Consider the following statements with
compel his king to marry Padmavati, the reference to Vedanga Kalpa:
daughter of the king of Magadha.

AMIGOS IAS 5
I. The second Vedanga is Kalpa, also known Gathasaptasati is attributed to a Satavahana
as the Veda Purusha's arms. king called Hala. Hence statement 2 is
II. The oldest Kalpa Sutras are those that correct.
have direct links to the Brahmanas and  The Satavahanas may have used gold as
Aranyakas in their contents. bullion, for they did not issue gold coins as
Which of the above given statements is/are the Kushans did. They issued mostly coins of
correct? lead, which are found in the Deccan. They
1) I only 2) II only also issued pot, copper, and bronze
3) Both I and II 4) Neither I nor II money.Hence statement 3 is not correct.
Answer: 3 20. Consider the following statements with
Explanation: respect to the Rig Vedic period:
• The second Vedanga is Kalpa (ritual), also I. Women slaves were prevalent during the
known as the Veda Purusha's arms. It is Rig Vedic period.
designed specifically for the right use of II. Land was a well-established property
Vedic scriptures. during the Rig Vedic age.
• The oldest Kalpa Sutras are those that III. Child marriage was widely prevalent
have direct links to the Brahmanas and during the Rig Vedic age.
Aranyakas in their contents. Which of the statements given above is/are
• The ritual (Kalpa), the main topic of the correct?
Brahmanas, was the first subject to get 1) I only 2) II and III only
systematic treatment in the Kalpa Sutras. 3) I and II only 4) I, II and III
19. Consider the following statements regarding Answer: 1
Satavahanas: Explanation:
I. The Satavahanas started the practise of • The Rig Vedic people may have
granting tax-free villages to Brahmanas. occasionally occupied pieces of land, but
II. The official language of the Satavahanas land did not form a well-established type
was Prakrit. of private property. Hence statement II is
III. They issued mostly gold coins. not correct.
Which of the statements given above is/are • In Rig Vedic times, we also notice the
correct? practise of levirate and widow remarriage.
1) I only 2) I and II only There are no examples of child marriage,
3) II and III only 4) I, II and III and the marriageable age in the Rig Veda
seems to have been 16–17. Hence
Answer: 2
statement III is not correct.
Explanation:
• Women slaves were prevalent during the
 The Satavahanas started the practise of Rig Vedic age. For example, when we hear
granting tax-free villages to Brahmanas and of gifts made for priests, they are stated in
Buddhist monks. The cultivated fields and terms of cows and women slaves and
villages granted to them were declared free never in terms of the measurement of
from molestation by royal policemen, land. Hence statement I is correct.
soldiers, and all kinds of royal officers. These
21. In the context of the Gupta period, the terms
areas therefore became small, independent
Bhuktis, Vishayas, and Vithis refer to
islands within the Satavahana
1) coins
kingdom.Hence statement 1 is correct.
2) administrative divisions
 The official language of the Satavahanas was
Prakrit. All inscriptions were composed in 3) administrative officials
this language and written in the Brahmi 4) books
script, as was the case in Asokan times. Some Answer: 2
Satavahana kings may have composed Explanation:
Prakrit books. One Prakrit text called

AMIGOS IAS 6
• The Guptas organised a system of II. Asoka is addressed as “Devanampiya” in
provincial and local administration. The the Aramaic language in the Lampaka
empire was divided into divisions inscription in Afghanistan.
(bhuktis), and each bhukti was placed III. James Princep deciphered both Brahmi
under the charge of an uparika. The and Kharosthi scripts used in the Asokan
bhuktis were divided into districts Inscription.
(vishayas), which were placed under the Which of the statements given above is/are
charge of vishayapati. In eastern India, the correct?
vishayas were divided into vithis, which 1) I only 2) II and III only
again were divided into villages.
3) I and III only 4) I, II and III
22. With reference to the later Vedic period,
Answer:4
which of the following statements is not
Explanation:
correct?
• Asokan inscriptions are found in two
1) The grant of land as a sacrificial fee was
places in Afghanistan: Kandahar and
not well established in the later Vedic
Lampaka.
period.
• The Kandahar inscription is bilingual,
2) The society came to be divided into four
written in Greek and Aramaic. Hence
varnas.
statement I is correct.
3) The concept of upanayana was prevalent
• In Lampaka, written in Aramaic, Asoka is
among all sections of society.
addressed as ‘Devanampiya’. Hence
4) Sacrifices and performing rituals became
statement II is correct.
the dominant mode of worshipping the
• James Prinsep, an officer in the mint of the
gods.
East India Company, deciphered Brahmi
Answer: 3
and Kharosthi, two scripts used in the
Explanation: earliest inscriptions and coins. Hence
• The concept of Upanayana (sacred thread statement III is correct.
ceremony) was limited to the upper 24. Match the following pairs with reference to
sections of society. Hence statement III is the officials of the Gupta administration:
incorrect.
I. Kumaramatyas: Through which the king
• The later Vedic society came to be divided kept close contact with the provincial
into four varnas called the brahmanas, administration.
rajanyas or kshatriyas, vaisyas, and sudras.
II. Vishyapatis: Governor of the district
The growing cult of sacrifices enormously
III. NagaraSreshtis: Provincial governors
added to the power of the brahmanas.
Hence statement II is correct. IV. Uparikas: The officers looking after the
city administration.
• In the later Vedic period, in addition to
cows, which were usually given as How many of the following pairs are correct?
sacrificial gifts, gold, cloth, and horses 1) Only one 2) Only two
were also givan. Sometimes the priests 3) Only Three 4) All Four
claimed portions of territory as dakshina, Answer: 2
but the grant of land as a sacrificial fee was Explanation:
not well established in the later Vedic • Kumaramatyas and Ayuktas – The king
period. Hence statement I is correct. maintained a close contact with the
• Sacrifices and rituals became more provincial administration through this
prevalent during the later Vedic period. class of officials. Hence, Piar I is correct.
Hence statement IV is correct. • Vishyapatis – Bhuktis were subdivided
23. Consider the following statements about into Vishyas or districts. They were
Asoka’s inscriptions: governed by Vishyapatis. Hence, Piar II is
I. The Kandahar inscription was written in correct.
Greek and Aramaic.

AMIGOS IAS 7
• NagaraSreshtis – the officers looking after  Syadvada is also known as the doctrine of
the city administration. Hence, Piar III is conditional predication. It is a method of
not correct. expressing the many-sidedness of reality
• Uparikas – Provinces in the Gupta Empire through the use of conditional statements.
were known as Bhuktis which are 27. Consider the following statements regarding
provincial governed by Uparikas. Hence, the legal system during the Gupta period:
Piar III is not correct. I. Civil and criminal laws were clearly
25. Consider the following statements with defined and demarcated during this
respect to the Kushan dynasty: period.
I. Kushans were considered to be one of the II. Laws were based on differences in varnas.
five branches of the Yuezhi tribe. III. Guilds of artisans and merchants were
II. Kanishka convened the fourth Buddhist governed by their own laws.
Council at Vaishali. Which of the statements given above is/are
Which of the above statement(s) is/are correct?
correct? 1) II and III only 2) I, II and III
1) I only 2) II only 3) I and II only 4) III only
3) Both I and II 4) Neither I nor II Answer: 2
Answer: 1 Explanation:
Explanation:  The judicial system was better developed
• The Kushan dynasty was administered under the Guptas than in earlier times.
from two capitals: Purushapura Several lawbooks were compiled during this
(Peshawar) and Mathura in northern period.
India.  For the first time, civil and criminal laws were
• In the 1st century CE, Kujula Kadphises clearly defined and demarcated. Theft and
founded the Kushan dynasty. adultery fall under criminal law. Disputes
• The Kushans are considered to be one of regarding various types of property came
the five branches of the Yuezhi tribe that under civil law. Elaborate laws were laid
lived on the Chinese frontier of Central down about inheritance. Hence statement I
Asia. Hence statement I is correct. is correct.
• Kanishka (127–151 AD) is considered the  During this period, many laws continued to
greatest ruler of the Kushan dynasty. He be based on differences in varnas. It was the
captured Patliputra and took the Buddhist duty of the king to uphold the law. The king
monk Asvaghosha with him to Peshawar. tried cases with the help of Brahmana
• Kanishka convened the fourth Buddhist priests. Hence statement II is correct.
Council in Kashmir. Hence statement II is  The guilds of artisans, merchants, and others
incorrect. were governed by their own laws. Seals from
26. Syadvada is a core theory and philosophy of Vaisali and from Bhita near Allahabad
which one of the following? indicate that these guilds flourished
1) Buddhism 2) Jainism exceedingly well in Gupta times. Hence
3) Sikhism 4) Vaishnavisim statement III is correct.
Answer: 2
Explanation: 28. Consider the following pairs of writers with
 Syadvada is a Jaina metaphysical doctrine their literature:
that states that all judgments are conditional I. Ashvagosha: Sariputra prakaran
and only hold true in certain II. Buddhagosa: Visuddhimagga
circumstances. The term "syadvada" comes III. Vasumitra: Mahavibasha sastra
from the Sanskrit word "syat," which means Which of the pairs given above is/are correctly
"perhaps" or "in some respects.” matched?
1) I only 2) I and II only

AMIGOS IAS 8
3) II and III only 4) I, II and III  It did not condemn the varna system, as
Answer: 4 Buddhism did. Hence statement II is not
Explanation: correct.
 Ashvaghosha wrote the Sanskrit drama  According to Mahavira, a person is born in a
Sariputraprakarana, which is about high or lower varna as a consequence of the
Sariputta, or Sariputra, a disciple of sins or virtues acquired by him in the
Buddha. Sariputraprakarana is considered previous birth. Mahavira looks for human
one of the earliest Sanskrit plays. Hence, Pair values even in a chandala. In his opinion,
I is correct. through pure and meritorious life, members
 Visuddhimagga is a 5th-century treatise by of the lower castes can attain liberation.
Buddhaghosa on Buddhist practise and  Jainism mainly aims at the attainment of
Theravāda Abhidhamma. Hence, Pair II is freedom from worldly bonds. It is not
correct. necessary to use any ritual to acquire such
 Vasumitra, a renowned philosopher of the liberation. It can be obtained through full
Kushan era, is considered to have written the knowledge and action. Full knowledge,
Mahavibhasa. Hence, Pair III is correct. action, and liberation are considered to be
the three gems or ratnas of Jaimsm. Hence
29. Which of the following philosophies in India
statement III is correct.
propounded the atom theory?
31. With reference to the Pandya dynasty during
1) Yoga 2) Nyaya
the Sangam age, consider the following
3) Sankhya 4) Vaisheshika
statements:
Answer: 4
I. The Pandyas are mentioned by
Explanation: Megasthenes in his texts and highlighted
 Vaisheshika is a form of atomism and an the matriarchal influence on the Pandya
orthodox school of Indian philosophy. It was society.
proposed by Maharishi Kanaad and explores II. The Pandya territory occupied the
causality, time, space, and the nature of southernmost part of the Indian
reality. peninsula, with its capital at
 Vaisheshika posits that the universe is made Kaveripattanam.
up of five substances: earth, water, air, fire, Which of the statements given above is/are
and space. Each of these five substances is of correct?
two types: paramāṇu and composite. 1) I only 2) II only
30. Consider the following statements with 3) Both I and II 4) Neither I nor II
respect to Jainism:
Answer: 1
I. It denied the existence of gods.
Explanation:
II. Unlike Buddhism, it condemned the Varna
 The Pandyas are first mentioned by
system.
Megasthenes, who says that their kingdom
III. It mainly aims at the attainment of was celebrated for pearls. He also speaks of
freedom from worldly bonds. it being ruled by a woman, which suggests
Which of the statements given above is/are some matriarchal influence in Pandya
correct? society. Hence statement I is correct.
1) I only 2) I and II only  The Pandya territory occupied the
3) III only 4) II and III only southernmost and south-eastern portion of
Answer: 3 the Indian peninsula, and it roughly included
Explanation: the modern districts of Tirunelveli, Ramnad,
 Jainism recognised the existence of the gods and Madurai in Tamil Nadu, with its capital
but placed them lower than the Jina. Hence at Madurai. Hence statement II is not
statement I is not correct. correct.
32. Kalakacharya-Katha, a very popular text,
belongs to which of the following sects?

AMIGOS IAS 9
1) Buddhism 2) Jainism famous for the headless, erect statue of
3) Ajivika 4) Vaishnavism Kanishka, whose name is inscribed on its
Answer: 2 lower part. Hence, Statement III is correct.
Explanation: 34. Consider the following pairs of Sangam texts
Kalakacharya Katha tells the storey of a great along with their themes:
Jain muni of the Shwetambar school called I. Tolkkappiyam: Philosophy and wise
Kalak. His sister and he joined the Jain maxims
monastic order at an early age. He was II. Tirukkural: Grammar and poetics
extremely knowledgeable, and she was III. Manimekalai: Adventures of Kovalan and
extremely beautiful. Even though she was a Madhavi's daughter.
nun, she was abducted by the king of Ujjaini. Which of the pairs given above is/are correctly
Hence, option (2) is correct. matched?
33. Consider the following statements regarding 1) I and II only 2) II and III only
the Gandhara and Mathura schools of art: 3) III only 4) I, II and III
I. The Mathura School of Art produced Answer: 3
sculptures of themes related to Buddhism Explanation:
only.
• Tolkappiyam is a Tamil text from the 1st–
II. The Kushan Empire played a major role in 4th century CE that is a treatise on
the development of the Gandhara school grammar and poetics. Hence statement I
of art. is not correct.
III. Kushana King, Kanishka's headless statue, • The Thirukkural, also known as Kural, is a
was produced in Mathura. Tamil-language text that is considered one
Which of the statements given above are of the greatest works on ethics and
correct? morality. Hence statement II is not
1) I and II only 2) II and III only correct.
3) I and III only 4) I, II and III • Manimekalai is a Tamil-Buddhist epic
Answer: 2 written in the 6th century CE by
Explanation: Kalavanikal Seethalai Sathanar. The epic is
 Early Jain Tirthankar images and portraits of about Manimekalai, the daughter of
kings are found at Mathura. Images of Kovalan and Madhavi, who later became a
Vaishnava (mainly Vishnu and his various Buddhist nun. It gives glimpses of the
forms) and Shaiva (mainly the lingas and development of the fine arts in the
mukhalingas) faiths are also found at Sangam Age. Hence statement III is
Mathura, but Buddhist images are found in correct.
large numbers. It may be noted that the 35. With reference to the "Yogachara" tradition
images of Vishnu and Shiva are represented of Buddhist philosophy, consider the
by their ayudhas (weapons). Hence, following statements:
Statement I is not correct. I. The world is built by consciousness and
 The Kushans brought together masons and has no more reality than a dream.
other artisans trained in different schools II. Yogachara philosophy was an essential
and countries. Indian craftsmen came into part of Hinayana Buddhism.
contact with the Greeks and the Romans, Which of the statements given above is/are
especially in the northwest frontier of correct?
Gandhara, giving rise to the Gandhara school 1) I only 2) II only
of art, which had a strong Greco-Roman 3) Both I and II 4) Neither I nor II
influence. Hence, Statement II is correct. Answer: 1
 The Gandhara art spread to many regions, Explanation:
thereby influencing various local schools like
• The Great Vehicle (the Mahayana) school
Mathura and Sarnath. Mathura School
of Buddhism chiefly has two philosophical
produced many Buddha images but was also

AMIGOS IAS 10
schools, the Madhyamika and the Which of the above given statements is/are
Yogachara.Hence statement II is not incorrect?
correct. 1) I only 2) II only
• The Yogachara (The Way of Union) or 3) Both I and II 4) Neither I nor II
Vijnanavadin school completely rejects Answer: 3
realism and maintains a thoroughgoing Explanation:
idealism, not allowing even the qualified
• Cheran Senguttuvan, a ruler from the 2nd
realism of the Madhyamikas. According to
century AD, introduced the Pattini cult to
this school, the world is built by
Tamil Nadu. The Pattini cult involves the
consciousness and has no more reality
worship of Kannagi as the ideal wife.
than a dream. The only reality is "Tathata"
Kannagi is the wife of Kovalan, and their
(suchness). Hence, statement I is correct.
story is the subject of the Tamil epic
36. Consider the following statements related to Silappadigalam. Hence statement I is
Harappan civilization: incorrect.
I. Harappans worshipped Earth as a fertility • The Pattini cult was prevalent during the
goddess. Sangam age. In medieval times, both
II. The Harappan script is largely Hindus and Buddhists assimilated the
alphabetical. Pattini cult.
III. The Harappan rulers were more • The Pugalur inscription is a 1st century AD
concerned with conquest than with inscription that mentions three
commerce. generations of Chera rulers. The
Which of the above-given statements is/are inscription is located near Karur and
incorrect? describes the Irumporai clan of Chera
1) I and II 2) I and III rulers. The inscription also records the
3) II and III 4) I, II and III construction of a rock shelter for Jains.
Answer: 3 Hence statement II is incorrect.
Explanation: 38. This Gupta ruler conquered places and
• In Harappa, numerous terracotta figurines unified a greater part of India under him. His
of women have been found. In one prestige and influence were so great that the
figurine, a plant is shown growing out of ruler of Sri Lanka, Meghavarman, sent a
the embryo of a woman. The image missionary seeking his permission to build a
probably represents the goddess of earth, Buddhist monastery at Gaya. His military
and therefore the Harappans worshipped greats are mentioned in the accounts of his
earth as a fertility goddess. Hence court poet, Harisena.
statement I is correct. Which of the ruler is being described in the
• The Harappan script is not alphabetical passage given above?
but largely pictographic. Hence statement 1) Skandagupta 2) Chandragupta II
II is incorrect. 3) Samudragupta 4) Chandragupta 1
• The Harappan rulers were more Answer: 3
concerned with commerce than with Explanation:
conquest, and Harappa was possibly ruled • The Gupta period in ancient India is
by a class of merchants. Hence statement referred to as the “Golden Age” because
III is incorrect. of the numerous achievements in the
37. With reference to the Sangam period, fields of arts, literature, science, and
consider the following statements: technology. It also brought about the
I. Karikala introduced the Pattini cult, or the political unification of the subcontinent.
worship of Kannagi. • Samudragupta (335–375 AD) of the Gupta
II. The pugalur inscription of the 1st century dynasty is known as the Napoleon of India.
A.D. has reference to three generations of Historian A. V. Smith called him so
Chola rulers. because of his great military conquests,

AMIGOS IAS 11
known from the 'Prayag Prashati' • The Vakatakas are noted for having been
(Allahabad Inscription) written by his patrons of the arts, architecture, and
courtier and poet Harisena, who also literature. Under the patronage of the
describes him as the hero of a hundred Vakataka king Harisena, the rock-cut
battles. He has been hailed for the Buddhist viharas and chaityas of the
unification of the greater part of the Ajanta Caves, which are now UNESCO
Indian subcontinent. He was a devotee of World Heritage Sites, were constructed.
Vishnu and a follower of the Brahmanical Hence statement III is correct.
religion. He granted permission to the • Pravarasena (c. 270–330 CE) is credited
Buddhist king Meghavarman (King of with establishing the true power and
Ceylon) to build monasteries in Bodh greatness of the Vakataka dynasty. He
Gaya. expanded the empire into Vidarbha and
• It is also mentioned that Samudragupta neighbouring regions of the Deccan, with
liked playing the lute and loved listening to Kanchanaka serving as his capital. Among
poems. He was titled ‘Kaviraj’ for his love the Vakataka kings, Pravarasena was the
for poems. Hence option (3) is the correct only one to adopt the title of Samrat,
answer. while the others preferred the more
39. With reference to the Vakatakas, consider humble title of Maharaja.
the following statements: 40. Consider the following statement with
I. The Vakatakas were contemporary to the reference to the Mauryan administration:
Mauryas in the north. I. Dhammamahatras were appointed for the
II. They ruled over the regions of northern administration of justice.
Maharashtra and Vidarbha. II. Rajukas were appointed to maintain
III. The Vakataka king, Harisena, contributed economic ties with neighboring countries.
to the rich heritage of the world heritage III. Shulkadhyakshas were officers in charge
site, Ajanta Caves. of royal income.
Which of the statements given above are Which of the statements given above are not
correct? correct?
1) I and III only 2) I and II only 1) I and II only 2) II and III only
3) II and III only 4) I, II and III 3) I and III only 4) I, II and III
Answer:3 Answer:1
Explanation: Explanation:
• The Vakataka dynasty was a Brahmin • Dhammamahatras: Dhammamahatras
dynasty of ancient India that emerged in were a group of officials appointed by
the Deccan region in the middle of the 3rd Emperor Ashoka of the Mauryan Empire
century CE. Vindhyashakti I was the to propagate his teachings on dharma and
founder of the Vakataka dynasty. to ensure their implementation
• The Vakatakas were contemporary of the throughout the empire. Hence statement
Guptas in northern India. Their history can I is not correct.
be reconstructed on the basis of a large • Rajukas: Rajukas was a group of officials in
number of copper-plate land grant the Mauryan Empire who were
charters issued to the Brahmanas. Even responsible for ensuring the proper
though they promoted Brahminism, they administration of justice and maintaining
were liberal in outlook and patronised law and order in the empire. Hence
Buddhism as well. Hence statement I is statement II is not correct.
not correct. • Shulkadhyakshas: Shulkadhyakshas were
• In peninsular India, the Vakatakas were a officers in charge of Royal income. The
local power that ruled over northern Shulkadhyakshas were appointed by the
Maharashtra and Vidarbha. Hence emperor and were responsible for
statement II is correct. supervising the collection of various taxes,

AMIGOS IAS 12
such as land tax, customs duty, tolls, and • Throughout his reign he possessed the
other levies. Hence statement III is Tondaimandalam region including the
correct. capital, Kanchi. After his death, the power
41. Hiuen Tsang visited Chalukya dynasty during of the Rashtrakutas declined.
the reign of which ruler? 43. Which of the following pair is incorrect
1) Pulakesin I 2) Pulakesin II regarding the type of land gifts in Chola
3) Vikramaditya 4) Kirthivarman II empire?
Answer: 2 1) Vellanvagai – land for non-Brahmana
Explanation: peasant proprietors
• The most important ruler of this dynasty 2) Brahmadeya – land gifted to Brahmanas.
was Pulakesin II. The Aihole inscription 3) Shalaboga – land for the maintenance of a
issued by him gives the details of his reign. school
He fought with the Kadambas of Banavasi 4) Devadana – land gifted to Jaina
and the Gangas of Mysore and established institutions.
his suzerainty. Answer: 4
• Durvinita, the Ganga ruler accepted his Explanation:
overlordship and even gave his daughter
in marriage to Pulakesin II. Another
notable achievement of Pulakesin II was
the defeat of Harshavardhana on the
banks of the river Narmada.
• He put a check to the ambition of Harsha
to conquer the south. In his first
expedition against the Pallavas, Pulakesin
II emerged victorious. But he suffered a
humiliating defeat at the hands of
Narasimhavarman I near Kanchi. 44. Consider the following statements regarding
• Subsequently, the Chalukya capital Vatapi the administration of the Vijayanagara
was captured and destroyed by the empire.
Pallavas. The most important event in the I. The king was advised by a council of
reign of Pulakesin II was the visit of Hiuen ministers known as Mahapratinidhi.
Tsang to his kingdom. II. Each province was under a governor
42. Who among the following defeated the known as ‘Mandaleshvara.’
Cholas at Takkolam? III. The Gauda was village headman.
1) Pulakesin II 2) Amoghavarsha I Which of the above statements is/are correct?
3) Krishna III 4) Vikramaditya I 1) I only 2) II only
Answer: 3 3) III only 4) I, II and III
Explanation: Answer: 4
• Among the successors of Amoghavarsha I, Explanation:
Krishna III (936 968 A.D.) was famous for
• The king was advised by a council of
his expeditions.
ministers (Mahapratinidhi) in the
• He marched against the Cholas and Vijayanagara Empire, which consisted of
defeated them at Takkolam. He marched the great nobles of the kingdom.
further south and captured Tanjore. However, the ultimate decisions were
• He went as far as Rameswaram and taken by the king himself. Hence,
occupied it for some time. statement I is correct.
• He built several temples in the conquered • The kingdom was divided into Rajyas or
territories including the Krishneswara Mandalam (provinces) below, which were
temple at Rameswaram.

AMIGOS IAS 13
Nadu (district), Sthala (sub-district) and 46. Arrange the following dynasties of Delhi
Grama (village). Sultanate in chronological order.
• Each province was under a governor I. Tughlaq Dynasty
known as ‘Mandaleshvara.’ Hence, II. Mamluk Dynasty
statement II is correct. III. Khilji Dynasty
• The Gauda was village headman and basic IV. Sayyid Dynasty
unit of administration. However, the Choose the correct option.
village self-government concepts were
1) I-II-III-IV 2) II-I-III-IV
considerably weakened under
3) III-II-I-IV 4) II-III-I-IV
Vijayanagara rule. Hence, statement III is
correct. Answer: 4
45. Consider the following statements regarding Explanation:
the importance of Bhakti movement • The Turks extended their conquest from
I. It provided an impetus for the the Punjab and Multan into the Ganga
development of regional languages. valley after the Battle of Tarain and even
captured Bihar and parts of Bengal.
II. The caste system was condemned by this
movement. • After that, for almost one hundred years
these Turkish invaders ruled over the vast
III. It gave equal importance to both men and
part of India and formed the Delhi
women.
Sultanate.
Which of the above given statements is/are
• The Turkish rulers were successful in
correct?
extending their rule over Malwa and
1) I only 2) I and II only
Gujarat and penetrate into the Deccan
3) II and III only 4) All the above and South India. The effects of the
Answer: 4 establishment of the Turkish rule resulted
Explanation: in far-reaching changes in society,
• The importance of the Bhakti movement administration, and cultural life of India.
was very great. Various preachers spoke • Dynasties of the Delhi Sultanate are:
and wrote in the regional languages. So, • Mamluk Dynasty (1206-1290 AD)
the Bhakti movement provided an • Khilji Dynasty (1290-1320 AD)
impetus for the development of regional
• Tughlaq Dynasty (1320-1414 AD)
languages such as Hindi, Marathi, Bengali,
• Sayyid Dynasty (1414-1451 AD)
Kannada, etc. Through these languages
they made direct appeal to the masses. • Lodi Dynasty (1451-1526 AD)
Hence, statement I is correct. 47. Consider the following statements
• As the caste system was condemned by I. Sufism stressed the elements of love and
the Bhakti saints, the lower classes were devotion as effective means of the
raised to a position of great importance. realisation of God.
Hence, statement II is correct. II. In Sufism, self-discipline was considered
• The importance of women in society was an essential condition to gain knowledge
also increased because the Bhakti of God by sense of perception.
movement gave equal importance to Which of the above given statements is/are
them. Hence, statement III is correct. correct?
• Moreover, the Bhakti movement gave to 1) I only 2) II only
the people a simple religion, without 3) Both I and II 4) Neither I nor II
complicated rituals. Answer: 3
• They were required to show sincere Explanation:
devotion to God. The new idea of a life of • Sufism stressed the elements of love and
charity and service to fellow people devotion as effective means of the
developed.

AMIGOS IAS 14
realisation of God. Hence, statement I is • Many buildings in Gujarathi and Bengali
correct. styles are found in this complex.
• Love of God meant love of humanity and • Gujarathi style buildings were probably
so the Sufis believed service to humanity built for his Rajput wives. The most
was tantamount to service to God. magnificent building in it is the Jama
• In Sufism, self-discipline was considered Masjid and the gateway to it called Buland
an essential condition to gain knowledge Darwaza or the Lofty Gate. The height of
of God by sense of perception. Hence, the gateway is 176 feet. It was built to
statement II is correct. commemorate Akbar’s victory over
• While orthodox Muslims emphasise Gujarat.
external conduct, the Sufis lay stress on • Other important buildings at Fatepur Sikri
inner purity. are Jodh Bai’s palace and Panch Mahal
• While the orthodox believe in blind with five storeys.
observance of rituals, the Sufis consider • During Akbar’s reign, the Humayun’s tomb
love and devotion as the only means of was built at Delhi, and it had a massive
attaining salvation. dome of marble. It may be considered the
• According to them one must have the precursor of the Taj Mahal.
guidance of a pir or guru, without which • Akbar’s tomb at Sikandara near Agra was
spiritual development is impossible. completed by Jahangir.
• Sufism also inculcated a spirit of tolerance • Nur Jahan built the tomb of Itimaddaulah
among its followers. Other ideas at Agra. It was constructed wholly of white
emphasised by Sufism are meditation, marble with floral designs made of semi-
good actions, repentance for sins, precious stones on the walls. This type of
performance of prayers and pilgrimages, decoration was called pietra dura. This
fasting, charity and suppression of method became more popular during the
passions by ascetic practices. reign of Shah Jahan.
48. Which of the following was not built by 49. Which of the following pair is incorrect
Akbar? regarding the administration of Marathas?
1) Agra fort 1) Sir-i-Naubat – Military commander
2) Panch Mahal 2) Amatya – Accountant general
3) Jama Masjid at Delhi 3) Waqenavis – Correspondence
4) Buland Darwaja 4) Sumanta – Master of ceremonies
Answer: 3 Answer: 3
Explanation: Explanation:
• Large scale construction of buildings • Shivaji was also a great administrator. He
started with the advent of Akbar. He built laid the foundations of a sound system of
many forts, and the most famous one was administration. The king was the pivot of
the Agra Fort. It was built in red the government. He was assisted by a
sandstone. His other forts are at Lahore council of ministers called Ashtapradhan.
and Allahabad. However, each minister was directly
• The climax of fort-building reached its responsible to Shivaji.
climax during the reign of Shah Jahan. * Peshwa – Finance and general
• The famous Red Fort at Delhi with its Rang administration. Later he became the
Mahal, Diwan-i-Am and Diwan-i-Khas was prime minister.
his creation. * Sar-i-Naubat or Senapati – Military
• Akbar also built a palace cum-fort complex commander, an honorary post.
at Fatepur Sikri (City of Victory), 36 * Amatya – Accountant General.
kilometres from Agra. * Waqenavis – Intelligence, posts and
household affairs.

AMIGOS IAS 15
* Sachiv – Correspondence. 4) Cotton, silk, salt Petre and opium.
* Sumanta – Master of ceremonies. Answer: 4
* Nyayadish – Justice. Explanation:
* Panditarao – Charities and religious • The staple commodities of export by the
administration. English East India Company from Bengal in
• Most of the administrative reforms of the middle of the 18th century were
Shivaji were based on the practices of the cotton, silk, salt Petre and opium.
Deccan sultanates. For example, Peshwa • After the Victory in Battle of Plassey and
was the Persian title. Buxar, the Company consolidated its rule
50. Many events happened in Europe caused in Bengal.
repercussion in India. The War of Austrian • Gradually, it became biggest exporter of
Succession took place from 1740 to 1748 AD. agricultural raw material which were used
The two main parties to this war were in production of finished goods in England.
1) British and Dutch These goods were also sold by the
2) French and British Company in European and Chinese
3) Dutch and Portuguese markets.
4) French and Dutch 53. Which of the following were the impacts of
the Revolt of 1857?
Answer: 2
I. Transfer of power from the East India
Explanation:
Company to the British Crown.
• The two main parties that were involved
II. A member of the British Cabinet was
in War of Austrian Succession were French
appointed Secretary of State for India.
and British.
III. The Governor-General of India was given
• The war came to an end with the Treaty of
the title of Viceroy.
Aix-la-Chapelle. According to the treaty
the English possessions in India were Choose the correct option.
returned, while the French got back their 1) I only 2) I and II only
North American possessions. 3) II and III only 4) All the above
51. British General, who defeated Hyder Ali in Answer: 4
war of Porto Novo was Explanation:
1) Captain Popham 2) Sir Hector Munro Impact of Revolt of 1857
3) Sir Eyre Coot 4) General Goddard • Revolt of 1857 was unprecedented and
Answer: 3 made Britishers feel seriously threatened
Explanation: which led to changes in administration
• In July 1781 AD, the British army under Sir and policies. The British Parliament passed
Eyre Coot defeated Hyder Ali at Porto a new Act in 1858 and transferred East
Novo. India Company to the British Crown in
order to ensure a more responsible
• The English war with Hyder Ali had started
management of Indian affairs. Hence,
in 1780 AD. Repeating his earlier exploits,
statement I is correct.
Hyder Ali inflicted one defeat after
another on the British armies and forced • A member of the British Cabinet was
them to surrender in large numbers. But appointed Secretary of State for India and
he was defeated in the battle at Porto made responsible for all matters related
Novo. to the governance of India. He was given a
council to advise him, called the India
52. The staple commodities of export by the
Council. Hence, statement II is correct.
English East India Company from Bengal in
the middle of the 18th century were. • The Governor-General of India was given
the title of Viceroy, i.e., a personal
1) Raw cotton, oilseeds, and opium.
representative of the Crown. Through
2) Sugar, salt, zinc, and lead
these measures, the British government
3) Copper, silver, gold, spices, and tea.

AMIGOS IAS 16
accepted direct responsibility for ruling II. Aihole inscription was composed by
India. Hence, statement III is correct. Pulakesin II.
• The change of responsibility to British Which of the above given statements is/are
crown was announced at darbar at correct?
Allahabad by Lord Canning in ‘Queen’s 1) I only 2) II only
Proclamation’ on 1st November 1858. So, 3) Both I and II 4) Neither I nor II
rule of East India Company came to an Answer: 4
end.
Explanation:
• As Britishers came to realise they had
Administration and Social Life under the
antagonised and made enemy of native
Chalukyas
rulers, they tried to win over them by
• The Chalukya administration was highly
promising them autonomy and respect to
centralized unlike that of the Pallavas and
their rights.
the Cholas. Hence, statement I is
• Indian annexed states and provinces were
incorrect.
brought under a single British
• Village autonomy was absent under the
paramountcy. The British officials found
Chalukyas.
army to be main cause of such revolt and
to prevent such reoccurrence, • The Chalukyas had a great maritime
reorganised it. power. Pulakesin II had 100 ships in his
navy. They also had a small standing army.
• The domination of the army by European
branch was fixed. The share of Europeans • The Badami Chalukyas were Brahmanical
in army was increased and fixed at 1 to 2 Hindus, but they gave respect to other
in Bengal and 2 to 5 in Madras and religions.
Bombay armies. • Importance was given to Vedic rites and
• The old policy of excluding Indian sepoys rituals. The founder of the dynasty
from officer ranks was still maintained. Pulakesin I performed the asvamedha
sacrifice.
54. The slogan “No taxation without
representation” was associated with which • A number of temples in honour of Vishnu,
of the following Act? Siva and other gods were also built during
this period.
1) Indian Councils Act, 1861
• Ravikirti, who composed the Aihole
2) Indian Councils Act, 1892
inscription was the court poet of Pulakesin
3) Indian Councils Act, 1909
II. Hence, statement II is not correct.
4) Government of India Act, 1919
56. Consider the following statements regarding
Answer: 2 Amoghavarsha
Explanation: I. His reign was famous for the cultural
• The reforms introduced through the development.
Indian Councils Act,1892 were severely II. He was a follower of Buddhism.
criticised at Congress sessions, where the
III. Kavirajamarga was written by him.
nationalists made no secret of their
Which of the above given statements is/are
dissatisfaction with them.
correct?
• Now, they demanded a majority of
1) I only 2) I and III only
elected Indians, and control over the
budget, i.e, the power to vote upon and 3) II and III only 4) All the above
amend the budget. They gave the slogan Answer: 2
“No taxation without representation”. Explanation:
55. Consider the following statements regarding • Govinda III’s successor Amoghavarsha I
the administration of the Chalukyas: (815- 880 A.D.) ruled for a long period of
I. The Chalukya administration was highly 64 years.
decentralized. • He had lost control over Malwa and
Gangavadi. Yet, his reign was popular for

AMIGOS IAS 17
the cultural development. Hence, 58. The controversy between Anglicists and
statement I is correct. Orientalists is associated with the
• He was a follower of Jainism. Jinasena was 1) Civil services
his chief preceptor. Hence, statement II is 2) Reorganisation of army
incorrect. 3) Education
• He was also a patron of letters and he 4) Judicial reforms
himself wrote the famous Kannada work, Answer: 3
Kavirajamarga. Hence, statement III is
Explanation:
correct.
• The controversy between Anglicists and
• He had also built the Rashtrakuta capital,
Orientalists is associated with the
the city of Malkhed or Manyakheda.
education. Anglicists were supporting
57. Which of the following events with respect to English language based modern education
the British East India Company was first to whereas Orientalists favoured traditional
occur? way of learning.
1) The Company established its first factory 59. The meaning of term Diku in the context of
in the south in Masulipatnam. the tribal movement is
2) Establishment of Fort William at Calcutta 1) Religious head of the tribals
3) William Hawkin’s arrival to India 2) Outsider in the tribal areas
4) The Mughal emperor Farrukhsiyar issued 3) Leaders of the tribal movement
a farman, called Magna Carta of the
4) Kind of guerrilla warfare
Company.
Answer: 2
Answer: 3
Explanation:
Explanation:
• The meaning of term Diku in the context
of the tribal movement is outsider. These
outsiders were composed of traders and
moneylenders who would come into the
forests to sell their goods not produced
within the forest.
• They were supported by the corrupt
British officials and were responsible for
exploitation of tribals.
60. With reference to the Chola period, the terms
‘Valangai and Idangai’ were
1) Two major sects of Shaivism
2) Two temples
3) Two taxes
4) Two major divisions of caste
Answer: 4
Explanation:
• Caste system was widely prevalent during
the Chola period. Brahmins and Kshatriyas
enjoyed special privileges.
• The inscriptions of the later period of the
Chola rule mention about two major
divisions among the castes – Valangai and
Idangai castes. However, there was
cooperation among various castes and
sub-castes in social and religious life.

AMIGOS IAS 18
• The position of women did not improve. • The Commission was not in favour of
The practice of ‘sati’ was prevalent among creating a separate state of Maharashtra
the royal families. for Marathi-speaking people and Gujarat
• The devadasi system or dancing girls for the Gujarati-speaking people. It
attached to temples emerged during this recommended Bombay to remain as it
period. was, that is, a bilingual province of
61. Consider the following statements with Gujarati and Marathi speakers. However,
respect to the Fazal Ali Commission (1953) on due to ongoing unrest, the State of
the reorganization of States: Bombay was finally divided into two
I. It accepted ‘One Language-One State” as separate states (Maharashtra and
the basis of the reorganization of the Gujarat) in 1960. Hence, Statement II is
States. incorrect.
II. It provided for the partition of Bombay • The Commission had rejected the creation
between Gujarati and Marathi-speaking of separate tribal states from the states of
people. Bihar and Assam. The Commission
observed that the separation of Bihar will
III. It rejected the demand to carve out tribal
affect the economy of the entire state.
states from Assam and Bihar.
The tribal region of the Chottanagpur
Which of the above statement(s) is/are
Plateau provided an important industrial
correct?
base. For Assam, the Commission had held
1) I only 2) II only that the demand by tribals for a separate
3) III only 4) I, II and III Hill State reflects the separatist pull of the
Answer: 3 extremist elements. Hence, Statement III
Explanation: is correct.
• In India, the most vigorous movement for 62. Consider the following events
linguistic autonomy was started by the I. First Round table conference
Telugu speakers. They demanded a II. Gandhi Irwin pact signed.
separate state for the Telugu-speaking III. Ramsay Mac Donald announces the
population as Andhra Pradesh. In 1952, Communal Awards
Potti Sriramulu started fast-unto-death to
IV. Adoption of Poorna Swaraj as goal
support his demand for a separate state
Arrange the above events in chronological
and died after a 56-day hunger strike for
order.
the cause. The government of India
acceded to the demand and appointed a 1) IV-I-II-III 2) I-IV-II-III
3-member commission in 1953, called as 3) I-IV-III-II 4) IV-I-III-II
Fazl Ali Commission to recommend Answer : 1
reorganization of the States. Explanation:
• The Commission accepted ‘linguistic 1929
homogeneity’ as the basis for • All Parties Muslim Conference formulates
reorganization. It accepted language as an the “fourteen points” under the
important factor conducive to leadership of Jinnah.
administrative convenience and • Bhagat Singh and Batukeshwar Dutt
efficiency. However, it rejected the theory dropped bomb in the central legislative
of ‘one language one state’. According to Assembly to protest against the Public
the Commission, unity, and integrity of Safety Bill.
India should be the primary consideration • Jatin Das dies in Jain after 64 days of
in the re-drawing of the borders. It fasting.
accepted language as not an exclusive or
• Lord Irwin’s announcement that the goal
binding principle, overriding all other
of British policy in India was the grant of
considerations. Hence, Statement I is
domination status.
incorrect.

AMIGOS IAS 19
• Lahore session of the Congress under Metcalfe Treaty with Maharaja Ranjit
Jawaharlal Nehru adopts the goal of Singh on 30th May 1809 AD.
complete independence (Poorna Swaraj) • This friendship was based upon the British
for India. fear of attack from Napoleon or Russia
1930 from the Northern Frontiers of the British
• Jawaharlal Nehru hoists the tricolour of Empire in India.
India on the banks of Ravi in Lahore. 64. Which of the following pair is incorrect
• First Independence Day observed. regarding the structures of Delhi Sultanate?
• The working committee of INC meets at 1) Adhai Din-ka Jhonpra – Qutub-ud-Din
Sabarmati and passed the Civil Aibak
Disobediences Movement with his epic 2) Alai Darwaja – Allauddin Khilji
Dandi march. 3) Tomb of Sikandar Lodi at Delhi – Ibrahim
• Mahatma Gandhi launches the Civil Lodi
Disobedience Movement with his epic 4) Nizamuddin Dargah – Firoz Shah Tughlaq
Dandi march. Answer: 4
• First Round table conference begins in Explanation:
London to consider the report of Simon
Commission for the future constitutional
set-up in India.

1931
• Gandhi Irwin pact signed. Civil
Disobedience movement suspended.
• Bhagat Singh, Sukh Dev and Raj Guru were 65. Which is the correct sequence of
executed (in Lahore Case). administrative units of Rashtrakuta’s in the
• Second Round Table conference begins descending order?
Mahatma Gandhi arrives in London to 1) Rashtras – Bhuktis – Vishayas – Villages
attend it. 2) Bhuktis – Rashtras – Vishayas – Villages
1932 3) Vishayas-Rashtras – Vishayas – Bhuktis –
• British Prime Minister Ramsay Mac Villages
Donald announces the Communal Awards 4) Rashtras – Vishayas -Bhuktis – Villages
giving separate electorate to Harijans get Answer: 4
reserved seats in place of separate
Explanation:
electorate.
• The Rashtrakuta Empire was divided into
• Gandhi’s fast unto death.
several provinces called rashtras under
• The Poona pact signed by which the the control of rashtrapatis.
Harijans get reserved seats in place of
• They were further divided into vishayas or
separate electorate.
districts governed by vishayapatis.
• Third Round table Conference begins in
• The next subdivision was bhukti consisting
London.
of 50 to 70 villages under the control of
63. In which year ‘Treaty of Friendship’ was bhogapatis.
signed between British and Maharaja Ranjit
• These officers were directly appointed by
Singh?
the central government. The village
1) 1805 2) 1845 administration was carried on by the
3) 1809 4) 1849 village headmen.
Answer: 3 • However, the village assemblies played a
Explanation: significant role in the village
• The British Government signed the ‘Treaty administration.
of Friendship’ also called as Minto-

AMIGOS IAS 20
66. Consider the following statements about • Ram Mohan Roy was given the title of Raja
Raja Ram Mohan Roy. by the titular Mughal Emperor of Delhi,
I. He wrote Gift to Monotheists and Akbar II whose grievances the former was
Precepts of Jesus. to present before the British king. Hence,
II. He was instrumental in the abolition of statement III is incorrect.
Sati in 1839. Contributions of Raja Ram Mohan Roy
III. He was given the title ‘Raja’ by the Mughal Towards Social Reforms
Emperor Bahadur Shah Zafar. • Raja Ram Mohan Roy organised a crusade
IV. His journals include Samvad Kaumudi and against practice of Sati.
Mirat-ul-Akbar. • He visited the cremation grounds,
Which of the above given statements is/are organised vigilance groups and filed
correct? counter petitions to the government
1) I and II only 2) II and III only during his struggle against Sati.
3) I and IV only 4) II and IV only • His efforts were rewarded by the
Government Regulation in 1829, which
Answer: 3
declared the practice of Sati a crime under
Explanation:
Lord William Bentinck. Hence, statement
Raja Ram Mohan Roy II is incorrect.
• He is greatly regarded as Father of Indian • He condemned polygamy and demanded
Renaissance and the Father of Bengal the right of inheritance and property for
Renaissance. women.
• During his early education, Ram Mohan • He assisted David Hare in forming Hindu
Roy studied Persian and Arabic at Patna. College and in 1825, he established
• Here he read the Quran, the works of Sufi Vedanta college.
mystic poets and the Arabic translation of • He was also a pioneer in Indian Journalism
the works of Plato and Aristotle. and brought out various journals,
• In Banaras, he studied Sanskrit and read newspapers in Bengali, Persian, Hindi and
Vedas and Upanishads. English.
• At the age of sixteen, he wrote a rational • When press censorship was relaxed by
critique of Hindu idol worship. Lord Hastings in 1819, Ram Mohan found
• He possessed a great love for his country three journals- The Brahmanical Magazine
and people and worked for their social, (1821); The Bengali weekly, Samvad
religious, political and intellectual Kaumudi (1821); and the Persian weekly,
upliftment. Mirat-ul-Akbar. Hence, statement IV is
• He wrote Gift to Monotheists (1809) and correct.
translated into Bengali the Vedas and the 67. Consider the following statements regarding
five Upanishads to prove his conviction to the Government of India Act 1935.
that ancient Hindu texts support I. It provided for establishment of All India
monotheism. Hence, statement I is Federation.
correct. II. The Act come into force in year 1936.
• In 1814, he set up the Atmiya Sabha (or III. The Act provided the ‘dyarchy’ system at
society of friends) in Calcutta to propagate the central level and introduced the
the monotheistic ideals of the Vedanta provincial autonomy.
and to campaign against idolatry, caste
Which of the above given statements is/are
rigidities, meaningless rituals, and other
correct?
social ills.
1) I only 2) I and III only
• Although he had great love for his Indian
3) II and III only 4) All the above
culture and traditions, he believed that
Western education alone could help to Answer: 2
regenerate the society. Explanation:

AMIGOS IAS 21
• Government of India Act, 1935 as this Act 1) Shankaracharya, Ramanuja, Jnaneshwara,
was passed by the British Parliament in Chaitanya
1935 and came into effect in 1937. Hence, 2) Ramanuja, Jnaneshwara, Chaitanya,
Statement II is incorrect. Shankaracharya
• Some of the key features of the Act were: 3) Jnaneshwara, Chaitanya, Ramanuja,
• the creation of a ‘Federation of India’ that Shankaracharya
consisted of two levels: a Central 4) Shankaracharya, Ramanuja, Chaitanya,
Executive and Parliament, and below it, Jnaneshwara
provinces, and princely states. Hence, Answer: 1
statement I is correct. Explanation:
• It discarded the ‘dyarchy’ system at the • Shankaracharya (788-820 AD) was a
provincial level and allowed for the Bhakti Saint and the proponent of Advaita
emergence of popularly elected provincial Philosophy. He played an important role in
legislatures. Hence, Statement III is reviving Hinduism when Buddhism was
correct. gaining popularity.
• Dyarchy was introduced at the central • Ramanuja (1017-1137 AD) was one of the
level, key subjects like defence and foreign earliest Bhakti Saint. He was Alvar Saint,
affairs were under the direct control of the and he advocated Vaishnavism as the best
Governor-General. means of attaining Salvation.
• There was a provision for joint sitting in • Jnaneshwara was a Bhakti saint from
cases of deadlock between the houses. Maharashtra. His time was from 1275 to
Three subject lists were introduced – 1296 AD. He wrote a commentary on the
Federal list, Provincial list and Concurrent Bhagavad Gita called Jnaneswari. His
list. other important works include Changdev
68. The governors of the provinces in Delhi Prasasti and Amrit Anubhava. He was
Sultanate were called the associated with the Krishnamargi sect.
1) Shikdars 2) Patwaris • Chaitanya (1486-1534 AD) was a Bhakti
3) Muqtis 4) Chaudris Saint from Bengal. He popularized the
Answer: 3 Krishna cult in Bengal and is the combined
Explanation: avatar of Radha and Krishna.
• The provinces under the Delhi Sultanate 70. The Ladh Khan temple, Durga temple and the
were called Iqtas. They were initially Huchimalligudi temple constructed by the
under the control of the nobles. But the Chalukyas are located at
governors of the provinces were called the 1) Aihole 2) Pattadakal
Muqtis or Walis. 3) Badami 4) Ajantha
• They were to maintain law and order and Answer: 1
collect the land revenue. The provinces Explanation:
were divided into Shiqs and the next The Chalukya temples may be divided into two
division was pargana. stages. The first stage is represented by
• The Shiq was under the control of shiqdar. the temples at Aihole and Badami. Among
• The pargana comprising a number of the seventy temples found at Aihole, four
villages was headed by amil. are important.
• The village remained the basic unit of the * Ladh Khan temple is a low, flat-roofed
administration. The village headman was structure consisting of a pillared hall.
known as Muqaddam or Chaudhri. * Durga temple resembles a Buddha
• The village accountant was called Patwari. Chaitya.
69. Which one of the following indicates the * Huchimalligudi temple.
correct chronological order of Bhakti saints? * The Jain temple at Meguti.

AMIGOS IAS 22
71. Consider the following events regarding the February 1948 resulting in 91% of the
integration of Princely States into the Indian electorates voting for accession to India.
Union: • Accession of Hyderabad was also a state
I. Instrument of Accession – Junagarh ruled by Muslim Nawab having a majority
II. Police action – Hyderabad Hindu population. The Nawab was not
III. Referendum – Jodhpur keen on the issue of accession to India.
Which of the above given pairs is/are correct? Operation Polo was conducted in
September 1948 and a contingent of
1) I only 2) I and II only
Indian troops was sent to Hyderabad. On
3) II only 4) II and III only
17th September 1948, the troops had full
Answer: 3 control over the State and Nizam
Explanation: announced on the radio to ‘live in peace
• The integration of Princely States was one and harmony with rest of the people of
of the biggest challenges faced by the India.’
nationalist leaders. With the combined 72. Which of the following is not correct
efforts of Sardar Patel and V.P. Menon, regarding the qualification to become a ward
most of the States had given their consent member of village council as per the
for accession to India. However, the states Uttarmerur inscription?
of Travancore, Bhopal, Jodhpur, Kashmir, 1) Ownership of at least one fourth veli of
Junagarh, and Hyderabad had a land.
troublesome history regarding their
2) Own residence.
integration into the Indian Union.
3) Between 40 to 60 years of age
• Accession of Jodhpur: At the time of
4) Knowledge of Vedas
independence, Jodhpur had a Hindu king
as well as a largely Hindu population. Answer: 3
Jinnah offered very lucrative terms to Explanation:
Jodhpur, and it was suspected the state to • The system of village autonomy with
join Pakistan. From the Indian side, Patel sabhas and their committees developed
intervened and additionally offered free through the ages and reached its
import of arms and adequate grains. culmination during the Chola rule.
Maharaja Hanwant Singh, thereafter, • Two inscriptions belonging to the period
signed the Instrument of Accession on of Parantaka I found at Uttiramerur
11th August 1947. provide details of the formation and
• Accession of Kashmir in October 1947, the functions of village councils. That village
Maharaja Hari Singh of Kashmir signed the was divided into thirty wards, and each
Instrument of Accession. It was against was to nominate its members to the
the backdrop of an invasion by Pashtun village council.
tribesmen from Pakistan’s Northwest • The qualifications to become a ward
Frontier Province (NWFP). After the member were:
signing of the Instrument, the Indian Army * Ownership of at least one fourth veli of
airlifted its troops to Kashmir and stopped land.
the invaders within two weeks. * Own residence.
• Accession of Junagarh had announced * Above thirty years and below seventy
that it would accede to Pakistan. Junagarh years of age.
had a Muslim Nawab ruling over chiefly * Knowledge of Vedas.
Hindu population (approximately 82%).
• However, certain norms of disqualification
However, geographically, its accession to
were also mentioned in the inscriptions.
Pakistan made little sense. To resolve the
They were:
matter, it was decided to conduct a
* Those who had been members of the
plebiscite. A referendum was held on 20th
committees for the past three years.

AMIGOS IAS 23
* Those who had failed to submit accounts means though the common man
as committee members. contented himself with one wife.
* Those who had committed sins. • Polygamy in shocking proportions
* Those who had stolen the property of prevailed among the kulin families in Uttar
others. Pradesh and Bengal.
73. The founding President of All India Kisan • Remarriage of widows was generally
Sabha was looked down upon though it prevailed in
1) Acharya Narendra Dev some places.
2) Swami Sahajanand Saraswati • Surprisingly the Peshwas imposed a tax
3) Bankim Mukerji called patdam on remarriage of widows.
4) Jayaprakash Narayan 76. Who was the author of the book
‘Moovarula’?
Answer: 2
1) Kalladanar 2) Pugalendi
Explanation:
3) Ottakuthar 4) Buddhamitra
• The founding President of All India Kisan
Sabha was Swami Sahajanand Saraswati. Answer: 3
It was formed in 1936. Explanation:
• NG Ranga, Ram Manohar Lohia, • The development of Tamil literature
Jayaprakash Narayan, Acharya Narendra reached its peak during the Chola period.
Dev, Bankim Mukerji were its other • Sivakasintamani written by
prominent members. Thiruthakkadevar and Kundalakesi
• It worked towards bringing the issues belonged to 10th century.
faced by the farmers by organising • The Ramayana composed by Kamban and
protest. the Periyapuranam or
• This organisation was associated with the Tiruttondarpuranam by Sekkilar are the
Communist Party of India. two masterpieces of this age.
74. When and where was All India Trade Union • Jayankondar’s Kalingattupparani
Congress established? describes the Kalinga war fought by
1) 1920 & Bombay 2) 1920 & Calcutta Kulotunga I.
3) 1925 & Bombay 4) 1925 & Calcutta • The Moovarula written by Ottakuthar
depicts the life of three Chola kings.
Answer: 1
• The Nalavenba was written by Pugalendi.
Explanation:
• The works on Tamil grammar like
• All India Trade Union Congress was
Kalladam by Kalladanar, Yapperungalam
formed in 1920 in Bombay. It was formed
by Amirthasagarar, a Jain, Nannul by
by NM Joshi with Lala Lajpat Rai as its
Pavanandhi and Virasoliyam by
President. It worked towards bringing all
Buddhamitra were the products of the
the labour movements happening in the
Chola age.
country under a single command.
77. Which of the following pair is incorrect?
75. In the context of Indian cultural history,
Patdam refers to a Organization Led By
1) Tax imposed on widow remarriage. 1) Arya Mahila Samaj
2) Tax imposed on religious ceremonies. : Pandita Rama Bai
3) Tax imposed on child marriages. 2) Ladies Society in Calcutta
4) Fine imposed to forgo heinous crimes. : Pandita Rama Bhai
Answer: 1 3) National Council of Women in India
Explanation: : Mehribai Tata
• Polygamy was common among ruling 4) Bharat Stree Mahamandali
princes, big zamindars and men of better : Sarla Devi Chaudhurani
Answer: 2

AMIGOS IAS 24
Explanation: Rani Sahiba of Sangli, Sarojini Naidu,
Women organizations led by women: Kamla Devi Chattopadhyaya, and Lady
• Arya Mahila Samaj – It was started by Dorab Tata among its members. This
Ramabai Saraswati(also known as Pandita organization was groundbreaking as it
Ramabhai) in Pune. Its main aim was to became the first women's organization in
provide education to women and to fight India with an egalitarian approach. AIWC
the age-old practices and discourage the operated both before and after India's
practice of child marriage. Later, she independence, advocating for
opened Sharada Sadan in Mumbai in 1889 fundamental principles of social justice,
for educating the child widows. integrity, equal rights, and opportunities.
• Bharat Mahila Parishad – It is the women’s Their mission aimed to ensure that every
wing of the National Social Conference individual, regardless of their birth or
(Part of Indian National Congress) in 1905 gender, had access to life's essentials
to improve the social conditions of through planned social distribution. AIWC
women. This parishad worked for the played a pivotal role in advancing gender
cause of dowry, child marriage and other equality and social reform in India during
evil customs. a critical period of its history.
• The Ladies Society in Calcutta – It was 78. Consider the following statements with
started in 1882 by Swarnakumari Devi, the reference to Non-Cooperation Movement
sister of Rabindranath Tagore. It worked I. The movement for the first time in history,
for imparting and educating livelihood acquired a real mass base with the
skills to the widows. Swarnakumari Devi participation of different sections of
was the first Indian Woman editor (the Indian society.
editor of the women’s journal Bharati). II. It transformed the Indian National
• Bharat Shree Mahila Mandal was Congress from a deliberative assembly
established by Sarala DevinChadhurani in into an organization for action.
1910 at Allahabad. It was first major Indian III. It marked the height of Hindu-Muslim
women’s organization set up by a woman. unity.
It worked for promotion of education for Which of the statements given above is/are
women, abolition of Purdhah system, and correct?
improving socio-economic-political 1) I only 2) I and II only
conditions of women. 3) II and III only 4) All the above
• In 1925, Mehribai Tata established the Answer: 4
National Council of Women in India, which Explanation:
served as the national branch of the
Non-Cooperation Movement (1921–1922)
International Council of Women. This
Significance
organization was dedicated to addressing
• The Indian Nationalist movement, for the
pressing issues faced by Indian women,
first time in history, acquired a real mass
including the removal of the purdah
base with the participation of different
system, the eradication of caste
sections of Indian society such as
differences, and the promotion of
peasants, workers, students and teachers,
education among women. Through its
women, merchants and so on. However,
efforts, the council aimed to empower
the big industrialists, capitalists, and
women, challenge social norms, and
zamindars still remained hostile. Hence,
foster gender equality in India during a
statement I is correct.
time when these issues were of
paramount concern. • Secondly, the movement witnessed the
spread of nationalism to the remotest
• The All-India Women's Conference (AIWC)
corners of the country.
was founded in 1927 and featured
prominent leaders such as Margaret • Thirdly, it transformed the Indian National
Cousins, Maharani Chimnabai Gaekwad, Congress from a deliberative assembly

AMIGOS IAS 25
into an organisation for action, as evident 80. Which of the following pair is incorrect
from the various programmes of the regarding the literature during the
movement. Hence, statement II is correct. Rashtrakuta period?
• Fourthly, it marked the height of Hindu- 1) Halayudha – Kavirahasya
Muslim unity which could be seen in the 2) Parsvabhudaya – Amoghavarsha
merger of the Khilafat movement with this 3) Adipurana – Gunabhadra
movement. Hence, statement III is 4) Ganitasaram – Viracharya
correct.
Answer: 2
• Finally, the movement demonstrated to a
Explanation:
remarkable degree the willingness and
• The Rashtrakutas widely patronized the
ability of the masses to endure hardships
Sanskrit literature. There were many
and make sacrifices in the cause of
scholars in the Rashtrakuta court.
national independence. Thus, though the
movement failed to achieve any of one of • Trivikrama wrote Nalachampu and the
its three main demands, it was, Kavirahasya was composed by Halayudha
nevertheless, a great step forward in the during the reign of Krishna III.
course of the Indian nationalist • The Jain literature flourished under the
movement. patronage of the Rashtrakutas.
79. When and where the Vaikom Satyagraha was Amogavarsha I, who was a Jain patronized
started? many Jain scholars. His teacher Jinasena
composed Parsvabhudaya, a biography of
1) 1922 & Kerala 2) 1924 & Kerala
Parsva in verses.
3) 1924 & Karnataka
• Another scholar Gunabhadra wrote the
4) 1922 & Karnataka
Adipurana, the life stories of various Jain
Answer: 2 saints.
Explanation: • Sakatayana wrote the grammer work
• Vaikom Satyagraha was a movement in called Amogavritti. The great
Travancore (modern-day Kerala) for mathematician of this period, Viracharya
temple entry of the depressed classes. It was the author of Ganitasaram.
took place near the Shiva Temple at • The Kannada literature saw its beginning
Vaikom, Kottayam district, Kerala during during the period of the Rashtrakutas.
1924- 25.
• Amogavarsha’s Kavirajamarga was the
• Vaikom Satyagraha started as a regional, first poetic work in Kannada language.
localized protest untouchability. It,
• Pampa was the greatest of the Kannada
however, grew in significance, and is
poets. His famous work was
considered a pivotal event in the history of
Vikramasenavijaya. Ponna was another
Kerala by historians, even leading to the
famous Kannada poet and he wrote
Temple Entry Proclamation of 1936. The
Santipurana.
heart of this protest was at the Vaikom
81. Which of the following pair is incorrect
Mahadeva Temple, where the upper caste
regarding the treaties and the respective
was permitted to use a public road around
regions?
it, but the others were not.
Treaty Region
• The movement began on 30th March
1924. At the Vaikom Mahadeva Temple, 1) Treaty of Sagauli : Nepal
there was a board which denied the entry 2) Treaty of Yandabo : Burma
of lower caste people (avarnas). The 3) Treaty of Lhasa : Bhutan
Satyagrahis made batches of three and 4) Treaty of Gandamak: Afghanistan
entered the temple. They were resisted Answer: 3
and arrested by the police. Explanation:
• Mahatma Gandhi came to participate in Treaty of Sagauli:
the Vaikom Satyagraha.

AMIGOS IAS 26
• It was signed between the Gurkha chiefs The Treaty of Lhasa:
of Nepal and the British Indian • It is officially the Convention Between
government that ended the Anglo- Great Britain and Tibet, was a treaty
Nepalese (Gurkha) War (1814–16). By the signed in 1904 between Tibet and the
treaty, Nepal renounced all claim to the British Empire, in Lhasa, the capital of
disputed Tarai, or lowland country, and Tibet, then under the administrative rule
ceded its conquests west of the Kali River of the Qing dynasty. It was signed
and extending to the Sutlej River. following the British expedition to Tibet of
• Nepal remained independent, but it 1903-1904, a military expedition led by
received a British resident with the status Colonel Francis Younghusband, and was
of an ambassador to an independent followed by the Anglo-Chinese
country rather than of the controlling Convention of 1906.
agent of the supreme government in an 82. A new policy of ‘paramountcy’ was initiated
Indian state. under the reign of, which of the following
Treaty of Yandabo: Governor-General?
• The first war with Burma was fought when 1) Lord Hastings 2) Lord Ripon
the Burmese expansion westwards and 3) Lord Curzon 4) Lord Dufferin
occupation of Arakan and Manipur, and Answer: 1
the threat to Assam and the Brahmaputra Explanation:
Valley led to continuous friction along the
• The policy of ‘Paramountcy’ was initiated
ill-defined border between Bengal and
under the reign of Lord Hastings (1813-
Burma, in the opening decades of the
1823 AD). Under this policy, the company
nineteenth century.
claimed that its authority was paramount
• It was signed during the tenure of Lord and supreme. Thus, he justified the
Amherst. The British expeditionary forces annexations of any Indian kingdom.
occupied Rangoon in May 1824 and
83. Which of the following pair is incorrect
reached within 72 km of the capital at Ava.
regarding the literature during the
Peace was established in 1826 with the
Vijayanagara period?
Treaty of Yandabo.
1) Parijatapaharanam – Nandi Timmana
Treaty of Sinchula:
2) Kavyalankara Sangraham – Pingali
• The Bhutan War is also known as the Duar
Suranna
War, and this ended in the defeat of the
3) Panduranga Mahatyamu – Tenali
Bhutanese army. Peace was brought by
Ramakrishna
the Treaty of Sinchula which was signed in
1865. Bhutan ceded territories in the 4) Harikathasaransamu – Allasani Peddana
Assam Duars and Bengal Duars, as well as Answer: 2
around 80,000 kilometers of Dewangiri Explanation:
(Deothang) to the British in return for an
annual subsidy of 50,000 rupees. It was
signed during the tenure of Sir John
Lawrence.
Treaty of Gandamak:
• It was signed during the tenure of Lord
Lytton. This treaty was signed after the
Second Anglo-Afghan War, 1879 provided
that: Amir conducted his foreign policy
with the advice of the Government of
India; a permanent British resident 84. Consider the following pairs with reference
stationed at Kabul the Government of to departments in Delhi Sultanate.
India gave Amir all support against foreign
aggression, and an annual subsidy.

AMIGOS IAS 27
I. Diwan-e-Risalat – It dealt with religious II. He advocated the path of self-surrender
matters. to God.
II. Diwan-e-Arz – Department of Agriculture Which of the above given statements is/are
III. Diwan-e-Amir-Kohi – Military department correct?
How many of the above pairs is/are correct? 1) I only 2) II only
1) Only one 2) Only two 3) Both I and II 4) Neither I nor II
3) All three 4) None Answer: 2
Answer: 1 Explanation:
Explanation: • In the twelfth century, Ramanuja, who
Major Departments of Delhi Sultanate was born at Sriperumbudur near modern
• Diwan-i-Risalat: It dealt with religious Chennai, preached Visishtadvaita.
matters, pious foundations, and stipends According to him God is Sagunabrahman.
to deserving scholars and men of piety. It Hence, statement I is incorrect.
was presided over by the chief sadr, who • The creative process and all the objects in
was generally the chief qazi. creation are real but not illusory as was
• The chief qazi was the head of the held by Sankaracharya. Therefore, God,
department of justice. The qazis soul, matter are real. But God is inner
dispensed civil law based on the Muslim substance, and the rest are his attributes.
law (sharia). The Hindus were governed by • He also advocated prabattimarga or path
their own personal laws. of self-surrender to God. He invited the
• Diwani-i-Arz: The Diwani-i-Arz or the downtrodden to Vaishnavism. Hence,
military department was the most statement II is correct.
important department of state, next to 86. The Battle of Plassey was fought between
the wazir. The head of this department 1) Mir Jafar and East India Company
was called the Ariz-i-Mumalik. 2) Shuja-ud-Daula and East India Company
• The office came into being for the first 3) Siraj-ud-Daulah and East India Company
time under Balban as a separate 4) Mir Khasim and East India Company
department. The Ariz was not the Answer: 3
Commander-in-Chief of the army, but it Explanation:
had responsibilities such as to recruit,
• In Battle of Plassey, the leading men of
equip and pay the army.
Nawab’s court, such as Mir Jafar, the Mir
• Diwan-i-Insha: It dealt with state Bakshi, Manik Chand, Amichand, Jagat
correspondence. There were a number of Seth intrigued with British army and did
other departments in addition to these. not join the battle.
The rulers posted intelligence agents
• It contributed in Nawab’s defeat. After the
called barids in different parts of the
Battle of Plassey, Robert Clive proclaimed
empire to keep them informed. Only a
Mir Jafar as the Nawab of Bengal thus
nobleman who enjoyed the fullest
making him a mere puppet in the hands of
confidence of the ruler was appointed the
British.
chief barid.
• Battle of Plassey was fought between
• Diwan-i-Amir-Kohi: The department was
Nawab of Bengal Siraj-ud-Daulah and the
related to agriculture, established by
East India Company.
Muhammad bin Tughlaq. It was started to
87. Consider the following statements about the
improve cultivation in the Doab region,
Tebhaga Movement in 1946-47:
which was facing acute drought and crop
failures. I. It was led by the Bengal Kisan Sabha.
85. Consider the following statements II. It was a clash between Bargadars and
Jotedars.
I. According to Ramanuja Charya, God is
Nirguna brahman. III. Floud Commission was established to
investigate the demands of the

AMIGOS IAS 28
sharecroppers, and it made suppress the peasants. Major clashes
recommendations in favour of them. ensued at a few places, the most
Which of the statements given above is/are important being the one at Khanpur in
correct? which twenty peasants were killed.
1) I only 2) I and II only Repression continued and by the end of
3) II and III only 4) All the above February, the movement was virtually
dead.
Answer: 2
• The Muslim League Ministry failed to
Explanation:
pursue the bill in the Assembly, and it was
Tebhaga Struggle
only in 1950 that the Congress Ministry
• The Tebhaga movement was significant passed a Bargadars Bill which
peasant agitation, initiated in Bengal by incorporated, in substance, the demands
the Bengal Kisan Sabha in 1946-47. Hence, of the movement.
statement I is correct.
• The main centers of the movement were
• In late 1946, the bargadars Dinajpur, Rangpur, Jalpaiguri,
(sharecroppers) of Bengal began to assert Mymensingh, Midnapore, and to a lesser
that they would no longer pay a half share extent 24-Parganas and Khulna.
of their crop to the jotedars (wealthy
• Initially, the base was among the Rajbansi
farmers) but only one-third and that
Kshatriya peasants, but it soon spread to
before division the crop would be stored
Muslims, Hajongs, Santhals, and Oraons.
in their khamars (godowns) and not that
• Among the important leaders of this
of the jotedars. They were encouraged by
movement were Krishnobinode Ray,
the fact that the Bengal Land Revenue
Abani Lahiri, Sunil Sen, Bhowani Sen, Moni
Commission, popularly known as the
Singh, Ananta Singh, Bhibuti Guha, Ajit
Floud Commission, had already made this
Ray, Sushil Sen, Samar Ganguli, and
the recommendation in its report to the
Gurudas Talukdar.
government. Hence, statement III is
incorrect.
• The Tebhaga movement, led by the Bengal 88. Consider the following statements regarding
Provincial Kisan Sabha, soon developed Kheda Satyagraha
into a clash between jotedars and I. It was the first non-cooperation
bargadars (sharecroppers) with the movement in Indian National movement.
bargadars insisting on storing the crop II. It was organized to suspend the revenue
inside their own godowns. Hence, assessment for the year 1919.
statement II is correct. III. Gandhi asked the farmers not to pay the
• The movement received a great boost in taxes.
late January 1947 when the Muslim Which of the statements given above is/are
League Ministry led by Suhrawardy correct?
published the Bengal Bargadars 1) I only 2) I and II only
Temporary Regulation Bill in the Calcutta 3) II and III only 4) All the above
Gazette on 22 January 1947. Encouraged Answer: 4
by the fact that the demand for tebhaga
Explanation:
could no longer be called illegal, peasants
Kheda Satyagraha (1918) – First Non-
in hitherto untouched villages and areas
Cooperation
joined the struggle. In many places,
peasants tried to remove the paddy • Because of drought in 1918, the crops
already stored in the jotedars’ godowns to failed in Kheda district of Gujarat.
their own, and this resulted in According to the Revenue Code, if the
innumerable clashes. yield was less than one-fourth the normal
produce, the farmers were entitled to
• The jotedars appealed to the
remission.
Government, and the police came in to

AMIGOS IAS 29
• The Gujarat Sabha, consisting of the • Ambedkar was against the caste-based
peasants, submitted petitions to the discriminations in society and advocated
highest governing authorities of the the Dalits to organise and demand their
province requesting that the revenue rights.
assessment for the year 1919 be • He promoted the education of Dalits and
suspended. Hence, statement II is correct. made representations to the government
• The government, however, remained in various capacities in this regard. He was
adamant and said that the property of the part of the Bombay Presidency Committee
farmers would be seized if the taxes were that worked with the Simon Commission
not paid. in 1925.
• Gandhi asked the farmers not to pay the • He established the Bahishkrit Hitakarini
taxes. Gandhi, however, was mainly the Sabha to promote education and socio-
spiritual head of the struggle. It was Sardar economic improvements among the
Vallabhbhai Patel and a group of other Dalits. He started magazines like
devoted Gandhians, namely, Narahari Mooknayak, Equality Janta and Bahishkrit
Parikh, Mohanlal Pandya and Ravi Shankar Bharat.
Vyas, who went around the villages, 90. The term Parados was widely used in the
organized the villagers and told them administration of Vijayanagara Empire. It
what to do and gave the necessary refers to.
political leadership. Patel along with his 1) Officer in-charge of royal household
colleagues organized the tax revolt which 2) Currency
the different ethnic and caste
3) Type of tax
communities of Kheda supported. Hence,
4) Fertile land
statement III is correct.
Answer: 2
89. The journals ‘Mooknayak’ and ‘Janta’ were
associated with which of the following Explanation:
person? • The term Parados was widely used in the
1) Swamy Vivekananda administration of Vijayanagara Empire
which refers to Currency.
2) Jawaharlal Nehru
• It had been estimated that while the
3) Dadabhai Nauroji
income of the kingdom was 12,000,000
4) Dr B R Ambedkar
paradoses, the Central Government got
Answer: 4 only half of the amount, during the
Explanation: Vijayanagara Empire.
Dr B R Ambedkar 91. Consider the following pairs.
• Popularly known as Baba Saheb. He was I. Sharab: Irrigation tax.
the Chairman of the Drafting Committee II. Khums: Land tax paid mainly by Hindus.
of the Constituent Assembly and is called
III. Kharaj: It was 1/6th of the booty captured
the ‘Father of the Indian Constitution’.
during war.
• He was a jurist and an economist. Born
IV. Ashraf: 1/10th of the gross produce on
into a caste that was considered
land held by Muslims.
untouchable, he faced many injustices and
How many of the above pairs is/are correct?
discrimination in society. He was born in
Mhow in the Central Provinces (modern- 1) Only one 2) Only two
day Madhya Pradesh) to a Marathi family 3) Only three 4) All Four
with roots in Ambadawe town of Answer: 2
Ratnagiri, Maharashtra. Explanation:
• He was a brilliant student and had Economy in the Sultanate Period
doctoral degrees in economics from the The new system of taxation was based on the
Columbia University and the London Quran. Four kinds of taxes sanctioned by the
School of Economics.

AMIGOS IAS 30
Quran were imposed namely kharaj, zakat, 93. Abul Hasan, Bishan Das and Ustad Mansur
jeziah and khums. were the painters during which of the
Types of Taxes following Mughal emperor?
• Zakat: To be paid by well to do Muslims for 1) Akbar 2) Humayun
helping the needy religious tax. 3) Shahjahan 4) Jahangir
• Jeziah: Tax levied on non-Muslims in Answer: 4
return of which they received protection Explanation:
of life and property and exemption from • Mughal paintings reached its climax
military services. during the reign of Jahangir. He employed
• Kharaj: Land tax paid mainly by Hindus a number of painters like Abul Hasan,
which was equal to 1/10th of the produce Bishan Das, Madhu, Anant, Manohar,
of land. Govardhan and Ustad Mansur.
• Khums: It was 1/6th of the booty captured • Apart from painting the scenes of hunting,
during war. battles and royal courts, progress was
• Abwabs: Excess taxes like house tax, made in portrait painting and paintings of
grazing tax etc. animals.
• Sharab: Irrigation tax. • Many albums containing paintings and
• Ashraf: 1/10th of the gross produce on calligraphy were produced during the
land held by Muslims. Mughal period. Later, the influence of
92. Consider the following European painting could be seen.
I. Kabir II. Guru Nanak 94. Bombay was taken over by the English East
III. Chaitanya IV. Ravidasa India Company from the
Who among the above was/were the disciples 1) Dutch 2) Danish
of Ramananda? 3) French 4) Portuguese
1) I and II 2) II and III Answer: 4
3) III and IV 4) I and IV Explanation:
Answer: 4 • Bombay was taken over by the English
Explanation: East India Company from the Portuguese.
In 1662 AD, the Portuguese gave the
• Ramananda was born at Allahabad. He
island of Bombay to king Charles II of
was originally a follower of Ramanuja.
England as dowry for marrying a
Later he founded his own sect and
Portuguese princess.
preached his principles in Hindi at Banaras
and Agra. • The king, then granted the control of
Bombay to the East India Company.
• He was a worshipper of Rama. He was the
Earlier, the naval battle between these
first to employ the vernacular medium to
countries begun in 1620 AD and the
propagate his ideas.
hostilities between these countries ended
• Simplification of worship and
in 1630 AD. Eventually, the Portuguese
emancipation of people from the
lost all their possessions in India except
traditional caste rules were his two
Goa, Diu and Daman.
important contributions to the Bhakti
95. Which of the following pair is correctly
movement.
matched?
• He opposed the caste system and chose
1) First Anglo-Mysore War - Hyder Ali was
his disciples from all sections of society
defeated.
disregarding caste.
2) Second Anglo-Mysore War - Hyder Ali
• His disciples were: a) Kabir, a Muslim
defeated the British.
weaver b) Raidasa, a cobbler c) Sena, a
barber d) Sadhana, a butcher e) Dhanna, a 3) Third Anglo-Mysore War - Tipu Sultan won
Jat farmer f) Naraharai, a goldsmith and g) the battle and did not cede his territory to
Pipa, a Rajput prince. the British.

AMIGOS IAS 31
4) Fourth Anglo Mysore War - Tipu was • The Company became a subordinate
defeated and was killed in the Battle. department of the State.
Answer: 4 • It introduced the Board of Control in
Explanation: charge of civil, military, and revenue
• First Anglo-Mysore War was held in 1767- affairs, and the Court of Directors to
69, which ended with the treaty of trading activity.
Madras. In this battle, Hyder Ali defeated • The Governor General was given a veto,
the British forces. and Madras and Bombay presidencies
• Second Anglo-Mysore War was fought in became subordinates.
1780-84 AD in which Hyder Ali was 97. Who is known for making this statement “a
defeated and this war ended with treaty single shelf of a good European library was
of Manglore. worth the whole native literature of India
• Third Anglo-Mysore War was held in 1790- and Arabia”?
92 AD and ended with the treaty of 1) Thomas Babington Macaulay
Srirangapatanam in, which Tipu Sultan 2) William Jones
ceded his territory to the British. 3) James Mill
• Fourth Anglo-Mysore War was fought in 4) Henry Thomas Colebrooke
1799 AD between the Britishers led by Answer: 1
Lord Wellesley and Tipu Sultan. In this Explanation:
battle, Tipu Sultan was killed and British
• Thomas Babington Macaulay is known for
annexed his Srirangapatam Fort.
making the statement “a single shelf of a
96. Consider the following statements. good European library was worth the
I. Pitt’s India Act established a system of whole native literature of India and
double government. Arabia”. This statement was his part of
II. The British Government was given speech to attack the orientalists.
supreme control over the Company’s • He was famous British historian and
affairs and its administration in India. politician of 19th century.
III. Pitt’s India Act was also known as the Act • He saw India as an uncivilised country that
of Settlement. need to be civilised. No branch of Eastern
Which of the above statements is/are correct? knowledge, according to him, could be
1) I only 2) I and II only compared to what England had produced.
3) II and III only 4) All the above 98. Who translated the Bhagavat Gita and
Answer: 2 Upanishads into the Persian language?
Explanation: 1) Abul Fazl 2) Dara Shikoh
• In a bid to rectify the defects of the 3) Inayat Khan 4) Naimatullah
Regulating Act of 1773, the British Answer: 2
Parliament passed the Amending Act of Explanation:
1781, also known as the Act of Settlement. • Jahangir’s autobiography, Tuzuk-i-
Hence, Statement III is incorrect. Jahangiri was famous for its style. He also
• The next important act was the Pitt’s India patronized many scholars like Ghiyas Beg,
Act of 1784. Naqib Khan and Niamatullah.
Features of the Act: • Shah Jahan also patronized many writers
• It distinguished between the commercial and historians like Abdul Hamid Lahori,
and political functions of the Company. author of Padshah Nama and Inayat Khan
• This act resulted in dual control of British who wrote Shah Jahan Nama.
possession in India by the British • His son Dara Shikoh translated the
government and company with final Bhagavat Gita and Upanishads into the
authority resting with the government. Persian language.
Hence Statement I&II are correct.

AMIGOS IAS 32
• Many historical works were written during 3) Indigo revolt 4) Kuki movement
the reign of Aurangazeb. Famous Answer: 3
dictionaries of the Persian language were Explanation:
also compiled during the Mughal period. The Bengali intellectual, Dinabandhu Mitra
99. Consider the following statements regarding wrote the popular play, Nildarpan, depicting
the Police system created by Lord Cornwallis the plight of Indigo cultivators.
I. The cities were placed in the charge of 102. Who had declared that ‘‘the extinction of all
Kotwals. native states of India is just a question of
II. The districts were divided into thanas, time’’?
each of which was put under the charge of 1) Lord Dalhousie 2) Lord Hardinge
Daroga. 3) Lord Lytton 4) Lord Irwin
III. The hereditary village policemen became Answer: 1
chowkidar.
Explanation:
Which of the above given statement(s) is/are
• Lord Dalhousie had declared that “the
correct?
extinction of all native states of India is
1) I only 2) I and II only just a question of time”.
3) II and III only 4) All the above • When Lord Dalhousie came out to India as
Answer: 4 the Governor General, he was from the
Explanation: beginning determined to extend direct
• To ensure good administration, Cornwallis British rule over as large area as possible.
created a permanent police force in India. • The above statements were uttered by
• Each district was divided into several him in this context only. The policy such as
Thanas, each of which was headed by a ‘Doctrine of Lapse’ was adopted by him in
Daroga. Hence, statement II is correct. pursuance of expansion of British rule in
• The hereditary village policemen became India.
chowkidar. Hence, statement III is correct. 103. Who devised the ‘Mahalwari Settlement’,
• The cities were placed under the Kotwals. which came into effect in 1822?
Hence, statement I is correct. 1) Captain Reed
100. Economically, one of the results of the British 2) John Shore
rule in India in the 19th century was the 3) Sir Thomas Munro
1) increase in the exports of Indian 4) Holt Mackenzie
handicrafts. Answer: 4
2) growth in the number of Indian owned Explanation:
factories. • The Mahalwari settlement was devised by
3) commercialisation of Indian agriculture. Holt Mackenzie in 1822 and it was
4) rapid increase in urban population. reviewed under Lord William Bentick in
Answer: 3 1833.
Explanation: • In this system, the land revenue was
• One of the results of British rule in India in collected by the village headmen, and it
the 19th century was the growth of was not fixed.
cultivation commercial crops and the raw • The peasants were the owner of the land
materials for the industries in Britain. and every peasant had to give his share of
• Different kinds of commercial crops such the revenue.
as tea, coffee, indigo, opium, cotton, jute, 104. The term ‘Mufti’ during the British period
sugarcane and oilseeds were introduced refers to
with different intentions. 1) A jurist of the Muslim community
101. Nildarpan was a popular play, it depicts responsible for expounding the law.
about 2) A land revenue officer appointed by the
1) Sanyasi revolt 2) Santhal rebellion British Government.

AMIGOS IAS 33
3) An officer responsible for controlling • Under the Yadava dynasty, effective
pilgrimage sites of Muslims under British military commanders, known as nayakas,
Government. were selected to head the provinces.
4) A judicial officer appointed by the British 106. Consider the following statements regarding
Government. the Home Rule League
Answer: 1 I. Annie Besant set up her league in
Explanation: September 1916.
• The meaning of the term Mufti is a jurist II. Tilak’s league covered Madras and rest of
belonging to the Muslim community India including Bombay.
whose duty was to expound the laws III. Tilak’s League was loosely organised as
which were to be administered by the compared to Annie Besant’s League.
Qazi. They were initially appointed in Which of the above given statements is/are
criminal courts (Fauzdari adalats). correct?
105. Consider the following pairs with respect to 1) I only 2) I and II only
Yadava Dynasty: 3) II and III only 4) All the above
I. Lakshmipati - the royal preceptor. Answer: 1
II. Mahapradhana - minister in charge of the Explanation:
treasury. Branches of Home Rule League
III. Mudraprabhu- officer in charge of seals. • Tilak and Besant set up separate leagues
Choose the correct code: to avoid friction.
1) I and III 2) III only • Tilak’s league was set up in April 1916 and
3) II and III 4) I and II it’s activities covered Maharashtra (except
Answer: 2 Bombay), Karnataka, Central Provinces
Explanation: and Berar.
• The administration of the Yadava Dynasty • It has six branches. Hence, statement II is
was similar to that of the other Deccan incorrect.
kingdoms. • Its demands were Swarajya, formation of
• The form of government was a hereditary linguistic states and education in
monarchy. vernacular languages.
• Yadavas had a strong bureaucracy in place • Annie Besant set up her league in
to support the administration. September 1916 in Madras and covered
• King used to consult his ministers prior to the rest of India (this included Bombay
issuing any land charter. also). Hence, statement I is correct.
• In ministry, Mahapradhana was a • It had 200 branches, was loosely
powerful minister who was usually in organised as compared to Tilak’s League
charge of a province or even a district. and George Arundale was its organising
Hence II pair is incorrect. secretary. Hence, statement III is
• Rajaguru was most likely the royal incorrect.
preceptor, counselling the monarch on • B P Wadia and C P Ramaswamy Aiyar also
religious matters. worked closely with these leagues. Major
• Lakshmipati was the minister in charge of Leaders in Leagues Leaders like Motilal
the treasury. Hence, I pair is incorrect. Nehru, Jawaharlal Nehru, Bhulabhai
Desai, Jinnah, Tej Bahadur Sapru, Lala
• Mudraprabhu, the officer in charge of
Lajpat Rai, CR Das, Madan Mohan Malviya
seals, Kusumarachanadhyaksha, the
joined leagues.
superintendent of floral arrangements,
the commander of hill forts, and the royal • Servants of India Society also joined the
hunter, are among the other royal officers movement. It was formed by Gopal
of the court. Hence III pair is correct. Krishna Gokhale.

AMIGOS IAS 34
• Muslims and Non-Brahmins from South Answer: 4
India did not join as they were under the Explanation:
impression that leagues were a Hindu • Paramaras were not involved in the
majority and dominated by high castes. tripartite struggle from the eighth to the
• Response of British Government to Home tenth century AD.
Rule Leagues • Tripartite struggle was between Pala
• The British Government dealt with high dynasty who dominated the Eastern India,
hands with leagues. Annie Besant and her Pratiharas who were in Western India and
companions, BP Wadia and George Rashtrakutas who dominated the Deccan
Arundale were arrested in June 1917. region and areas in North and South as
• Opposing the arrest, Sir Subramaniya well.
Aiyar renounced his Knighthood and Tilak 109. In the context of Hoyasala’s administration,
began passive resistance against this Gavundas are:
action of the government. 1) Treasurer 2) Chief Justice
107. Which of the following was not a 3) Revenue official
recommendation of the Simon Commission? 4) Commander-in-chief
1) The abolition of dyarchy. Answer: 3
2) The Governor-General was to have Explanation:
complete power to appoint the members
• The kingdom was divided into provinces or
of the Cabinet.
regions, each administered by local
3) Separate communal electorates be governors or chiefs who were appointed
stopped. by the king.
4) Accepted the idea of federalism. • Revenue officials, known as Gavundas,
Answer: 3 were responsible for assessing and
Explanation: collecting taxes from the agricultural
Recommendations of Simon Commission sector. Hence option 3 is correct.
• It proposed the abolition of dyarchy and • Senior ministers, called Pancha
the establishment of representative Pradhanas, ministers responsible for
government in the provinces. foreign affairs, designated Sandhivigrahi,
• It rejected parliamentary responsibility at and the chief treasurer, Mahabhandari or
the centre. The Governor-General was to Hiranyabhandari conducted top-level
have complete power to appoint the government affairs.
members of the Cabinet. • Dandanayakas led the armies while
• It recommended that separate communal Dharmadhikari served as the chief justice
electorates be retained. of the Hoysala court.
• It accepted the idea of federalism but not • Local officials, known as Nyayamurtis or
in the near future. Nyayadhishas, presided over the local
• It suggested that a Consultative Council of courts and helped in the administration of
Greater India should be established which justice.
should include representatives of both the 110. ‘Kudavolai system’ during the Cholas refers
British provinces as well as princely states. to
• It also suggested that the Indian army 1) System of nomination of members to the
should be Indianised, though British forces village council
must be retained. 2) Land grants system.
108. Which among the following kingdoms was 3) System of allocating financial grants to
not involved in the tripartite struggle from villages
the eighth to the tenth century AD? 4) Taxation system
1) Palas 2) Rashtrakutas Answer: 1
3) Pratiharas 4) Paramaras Explanation:

AMIGOS IAS 35
• The system of village autonomy with I. It was built during the period of
sabhas and their committees developed Ganapatideva.
through the ages and reached its II. It is a Trikutalaya.
culmination during the Chola rule. III. The three deities in the temple are Rudra,
• Two inscriptions belonging to the period Vasudev and Surya.
of Parantaka I found at Uttiramerur Choose the correct option.
provide details of the formation and 1) I only 2) I and II only
functions of village councils. That village
3) II and III only 4) All the above
was divided into thirty wards, and each
Answer: 3
was to nominate its members to the
village council. Explanation:
• The qualifications to become a ward • The Anumakonda Thousand pillar temple
member were: was built during the period of Rudradeva
in 1163 A.D. Hence, statement I is
 Ownership of at least one fourth veli of
incorrect.
land.
• This is a Trikutalaya, Rudra, Vasudev and
 Own residence.
Surya are main deities, declarations of
 Above thirty years and below seventy
Kakatiyas Sovereignity is supported by the
years of age.
inscriptions of 1163 A.D, found in the
 Knowledge of Vedas. temple. Hence, statement II and II are
• However, certain norms of disqualification correct.
were also mentioned in the inscriptions. 112. Which of the following statements is/are
They were: correct
 Those who had been members of the I. Magallu inscription talks about donation
committees for the past three years. of the village Magallu to a Brahmin by
 Those who had failed to submit accounts Kakartya Gundana.
as committee members. II. Magallu inscription was issued by
 Those who had committed sins. Rudradeva
 Those who had stolen the property of Choose the correct option.
others. 1) I only 2) II only
• From the persons duly nominated, one 3) Both I and II 4) Neither I nor II
was to be chosen for each ward by
Answer: 1
kudavolai system for a year. The names of
Explanation:
eligible persons were written on palm-
leaves and put into a pot. • Magallu inscription which was issued by
Eastern Chalukyan king Danarnava talks
• A young boy or girl would take out thirty
donation of the village Magalluto a
names each for one ward. They were
Brahmin by Kakartya Gundana. Hence,
divided into six variyams such as
statement I is correct, and statement II is
samvatsaravariyam, erivariyam, thotta
incorrect.
variyam, pancha variyam, pon variyam
and puravuvari variyam to take up six 113. Consider the following statements related to
different functions of the village Bhillama of Yadava dynasty:
administration. I. Bhillama was referred to as "Chakravartin
• The committee members were called Yadava" in Mutugi inscription.
variyapperumakkal. They usually met in II. The city of Chandradityapura was founded
the temple or under a tree and passed by him.
resolutions. The number of committees III. He patronised the scholar Bhaskara, who
and ward members varied from village to wrote Siddhanta Siromani.
village. Which of the above given statements is/are
111. Consider the following statements about correct?
Thousand pillar temple during Kakatiyas. 1) I and II 2) II and III

AMIGOS IAS 36
3) I and III 4) I, II and III jewelry worn by the deities, are intricately
Answer:3 carved. Hence statement II is correct.
Explanation: • Pierced stone window screens are popular
• Bhillama forced Ballala to retreat around in Hoysala temples. Most Hoysala temples
1187, conquered the former Chalukya have a plain covered entrance porch
capital Kalyani, and declared himself a supported by lathe turned (circular or bell-
sovereign ruler. shaped) pillars. Larger Hoysala temples
• He then founded the city of Devagiri, often include an exterior mantapa
which became the new Yadava capital. servicing the outer hall, which connects to
Hence statement II is incorrect. an inner tiny enclosed mantapa and
shrines.
• Chandradityapura was founded by Dridha
Prahara. • The walls and stairs of the temple
followed Zigzag pattern. Hence statement
• The Mutugi inscription is a 1189-90 CE
III is incorrect.
(1111 Shaka) record that refers to
Bhillama as "Chakravartin Yadava". The 115. Consider the following statements with
inscription also claims that Bhillama respect to Reddi Dynasty:
defeated the kings of Anga, Vanga, I. It was founded by Prolaya Vema Reddi.
Nepala, and Panchala. Hence statement I II. Addanki was the initial capital of the
is correct. kingdom.
• The inscription also mentions that III. Kataya Vema was the Reddy king who
Bhillama patronized the scholar levied tax on the delivery of the child
Bhaskaracharya, who wrote the Siddhanta (Purutu Pannu).
Siromani. Hence statement III is correct. Which of the above given statements is/are
• The Yadavas was the first major dynasty to correct?
adopt Marathi as an official language. 1) I and III 2) II only
114. Consider the following statements with 3) I and II 4) I, II and III
respect to Hoyasala architecture: Answer: 3
I. They use soft soapstone (Chorite schist) as Explanation:
main building material. • It was during this chaotic period in Andhra
II. The temples are generally built on stellate history that Prolaya Vema Reddi
(star-shaped) platforms. established the Reddi kingdom in 1325. He
III. The walls and stairs of the temple was revered by the title of Apratima-
followed uniform pattern. Bhudana-Parasuram. Hence statement I is
Which of the above given statements is/are correct.
correct? • The initial capital of the kingdom was
1) I and II 2) II only Addanki. Later, it was moved to Kondavidu
3) II and III 4) I, II and III and a subsidiary branch was established at
Answer: 1 Rajahmundry. Hence statement II is
correct.
Explanation:
• The Reddis were known for their
• The Hoysalas primarily used soapstone
fortifications.
(chloritic schist) as their primary building
material. The malleable stone makes it • Racha vema was the Reddy king who
easy to carve and has contributed to many levied tax on the delivery of the child.
intricate sculptures on the temple walls. Heavy taxes by Racha Vema Reddi made
Hence statement I is correct. him highly unpopular. Hence statement III
is incorrect.
• The temples are generally built on stellate
(star-shaped) platforms. Both the interior 116. With reference to Subhash Chandra Bose,
and exterior walls, even the pieces of consider the following statements:
I. The newspaper ‘Swaraj’ was started by
Subhash Chandra Bose.

AMIGOS IAS 37
II. Jawaharlal Nehru constituted a National 4) Rajendra Prasad
Planning Committee under the Answer: 3
chairmanship of Subhash Chandra Bose. Explanation:
Which of the statements given above is/are • Sardar Vallabhbhai Patel chaired the
correct? Congress in 1931 when it passed the
1) I only 2) II only "Fundamental Rights and Economic
3) Both I and II 4) Neither I nor II Policy" resolution.
Answer: 1 • Patel was an active participant in the
Explanation: freedom movement and is known as the
• Subhash Chandra Bose was one of the "Iron Man of India".
most celebrated freedom fighters and • He worked against caste discrimination,
charismatic influencers of the youth of untouchability, and alcohol consumption,
India. and for women's emancipation.
• In 1921, Bose worked under Chittaranjan 118. Which of the following statements are
Das, a powerful politician in Bengal. He correct in relation to the instructions given by
worked as the editor for Das’s newspaper, Gandhiji to the nation during Quit India
“Forward”, and later started his own Movement, 1942?
newspaper, “Swaraj”. I. Government servants were asked to
• He joined the Indian National Congress in resign and openly declare their allegiance
1927 and supported the idea of complete to Congress.
independence (Purna Swaraj). With his II. Soldiers were asked not to leave their
support, Jawaharlal Nehru passed the posts, but to refuse to fire on the people.
resolution of Purna Swaraj in 1929. III. Gandhiji asked Princely states to accept
• He was elected as the Congress President the sovereignty of their own people rather
in 1938, when the annual session was than paying homage to foreign power.
convened at Haripur in Bengal. IV. Peasants should refuse to pay the land
• In 1938, the President of the Indian revenue and ryots should outrightly refuse
National Congress, Subhash Chandra to pay any tax to any zamindar.
Bose, constituted a National Planning Select the correct answer using the code given
Committee under the chairmanship of below.
Jawaharlal Nehru. 1) I, II and IV only 2) I, II, III and IV
• He set up the Committee to ensure equal 3) II and III only 4) I, III and IV only
distribution of wealth among the diverse Answer: 3
sections.
Explanation:
• He resigned in 1939 due to differences
• Gandhiji's speech at the All-India Congress
with Mahatma Gandhi and set up the All-
Committee meeting at Gowalia Tank in
India Forward Bloc.
Bombay contained specific instructions
• In 1942, he earned the title ‘Netaji’, in for different sections of the people.
Germany from the Indian soldiers of the
• Government servants would not yet be
Azad Hind Fauj.
asked to resign, but they should openly
• He is also accredited to be the first man to declare their allegiance to Congress.
call Mahatma Gandhi “Father of the Hence, statement I is not correct.
Nation”, in his address from Singapore.
• Soldiers were also not to leave their posts,
117. Under Whose chairmanship, “Fundamental but they were to ‘refuse to fire on our own
Rights and Economic Policy” resolution was people. Hence, statement II is correct.
passed by the Congress in 1931?
• The princes were asked to ‘accept the
1) Jawaharlal Nehru sovereignty of your own people, instead
2) Dr. B. R. Ambedkar of paying homage to a foreign power.’ And
3) Vallabhai Patel the people of the Princely States were

AMIGOS IAS 38
asked to declare that they were part of the * the return of confiscated lands not yet
Indian nation and that they would accept sold to third parties, and
the leadership of the princes, if the latter * lenient treatment for those government
cast their lot with the People, but not employees who had resigned.
otherwise. Hence, statement III is correct. • The Government also conceded the right
• Students were to give up studies if they to make salt for consumption to villages
were sure they could continue to remain along the coast, as also the right to
firm independence was achieved. peaceful and non-aggressive picketing.
• Peasants ‘who have the courage and are • Congress demand for a public inquiry into
prepared to risk their all’ should refuse to police excesses was not accepted, but
pay the land revenue. Tenants were told Gandhiji’s insistent request for an inquiry
that Congress holds that the land belongs was recorded in the agreement. The
to those who work on it and to no one Congress, on its part, agreed to
else. Where the zamindari system discontinue the Civil Disobedience
prevails, if the zamindar makes common Movement. It was also understood that
cause with the riot, his portion of the Congress would participate in the next
revenue, which may be settled by mutual Round Table Conference.
agreement, should be given to him. But if • Hence option (3) is the correct answer.
a zamindar wants to side with the 120. Which of the following leaders founded the
Government, no tax should be paid to "Harijan Sevak Sangh" in 1932?
him. These instructions were not actually
1) Mahatma Gandhi 2) Dr. B.R. Ambedkar
issued because of the preventive arrests,
3) Jawahar Lal Nehru 4) K. Kelappan
but they do make Gandhiji’s intentions
clear. Hence, statement IV is not correct. Answer: 1
Explanation:
119. Which of the following were part of the • Harijan Sevak Sangh was founded by
Gandhi-Irwin Pact of 1931? Mahatma Gandhi in 1932 when he left
Sabarmati and decided to settle in
I. Right to make salt for personal
Wardha till Swaraj and eradication of
consumption
untouchability was achieved. He
II. Release of political prisoners who were
undertook intensive "Harijan Tour"
not convicted for violence.
traveling across the country and
III. Public inquiry into police excesses propagating the removal of untouchability
IV. Right to peaceful picketing in all its forms and practices. He collected
Which of the above given statements is/are money under this organization.
correct. • After the Second Round Table Conference,
1) I and III only 2) III and IV only the British government agreed to give
3) I, II and IV only 4) I, II, III and IV Communal Award to the depressed
Answer: 3 classes on the request of B. R. Ambedkar.
Explanation: Gandhi opposed the government's
• 5 March 1931, the Gandhi-Irwin Pact was decision which he considered would
signed by Gandhiji on behalf of the divide the Hindu society and subsequently
Congress and by Lord Irwin on behalf of went on to the indefinite fast in Yerwada
the Government. Jail. He ended his fast after signed Poona
Pact with Ambedkar on 24 September
• The pact placed the Congress on an equal
1932. On 30 September, Gandhi founded
footing with the Government. The terms
All India Anti Untouchability League, to
of the agreement included.
remove untouchability in the society,
* the immediate release of all political
which later renamed as Harijan Sevak
prisoners not convicted for violence,
Sangh ("Servants of Untouchables
* the remission of all fines not yet collected, Society").

AMIGOS IAS 39
• At the time industrialist, Ghanshyam Das
Birla was its founding president with
Amritlal Takkar as its Secretary.

AMIGOS IAS 40

You might also like